07.05.2024

Скорость магнитной индукции: «Как вычисляется скорость изменения индукции магнитного поля?» – Яндекс.Кью

Содержание

Репетитор-онлайн — подготовка к ЦТ


Пример 13. Проводящий контур, имеющий форму квадрата со стороной 20 см, помещен в однородное магнитное поле с индукцией 45 мТл. Плоскость контура составляет угол 30° с направлением силовых линий поля. За 0,15 с контур поворачивают таким образом, что его плоскость устанавливается перпендикулярно силовым линиям поля. Найти среднее значение ЭДС индукции, возникающей в контуре при его повороте в магнитном поле.


Решение. Появление ЭДС индукции в контуре вызвано изменением потока вектора индукции, пронизывающего плоскость квадрата, при повороте контура в магнитном поле.

Поток индукции магнитного поля через площадь квадрата определяется формулами:

  • в первом положении контура (до поворота)

Ф1 = BS cos α1,

где B — модуль индукции магнитного поля, B = 45 мТл; S — площадь квадрата, S = a
2; a — сторона квадрата, a = 20 см; α1 — угол между направлениями вектора магнитной индукции и вектора нормали (перпендикуляра) к плоскости квадрата в первом положении контура, α1 = = 90° − 30° = 60°;

  • во втором положении контура (после поворота)

Ф2 = BS cos α2,

где α2 — угол между направлениями вектора магнитной индукции и вектора нормали (перпендикуляра) к плоскости квадрата во втором положении контура, α2 = 0°.

Изменение потока вектора индукции магнитного поля определяется разностью


ΔФ=Ф2−Ф1=BScos0°−BScos60°=BS2.

Среднее значение ЭДС индукции, возникающей в контуре при его повороте в магнитном поле:


〈ℰi〉=|ΔФΔt|=−BS2Δt=Ba22Δt,

где ∆t — интервал времени, за который происходит поворот контура, ∆t = 0,15 с.

Расчет дает значение:


〈ℰi〉=45⋅10−3⋅(20⋅10−2)22⋅0,15=6,0⋅10−3 В=6,0 мВ.

При повороте контура в нем возникает ЭДС индукции, среднее значение которой равно 6,0 мВ.

Формула ЭДС индукции, E

Закон Фарадея – Максвелла для электромагнитной индукции

Основной формулой, которая определяет ЭДС индукции, является закон Фарадея – Максвелла, больше известный как основной закон электромагнитной индукции (или закон Фарадея). Этот закон утверждает, что ЭДС индукции в контуре, находящемся в переменном магнитном поле, равна по величине и противоположна по знаку скорости изменения магнитного потока () через поверхность, которую ограничивает данный контур:

   

где – скорость изменения магнитного потока. Полная производная в законе (1) охватывает весь спектр причин изменения магнитного потока через поверхность контура. Знак минус в формуле (1) соответствует правилу Ленца. Формула (1) для ЭДС индукции записана для системы СИ.

В случае равномерного изменения магнитного потока формулу ЭДС индукции можно записать как:

   

Частные случаи формул ЭДС индукции

Если контур содержит N витков, которые соединяются последовательно, то ЭДС индукции вычисляют как:

   

где – потокосцепление.

При движении прямолинейного проводника в однородном магнитном поле в нем возникает ЭДС индукции, которая равна:

   

где v – скорость движения проводника; l – длина проводника; B – модуль вектора магнитной индукции поля; .

При вращении с постоянной скоростью в однородном магнитном поле плоского контура вокруг оси, которая лежит в плоскости контура в нем возникает ЭДС индукции, равная:

   

где S – площадь, которую ограничивает виток; – поток самоиндукции витка; — угловая скорость; () – угол поворота контура. Следует учесть, что формула (5) справедлива, если ось вращения составляет прямой угол с направлением вектора внешнего поля .

Если во вращающейся рамке имеется N витков и самоиндукцией рассматриваемой системы можно пренебречь, то:

   

В стационарном проводнике, который находится в переменном магнитном поле, ЭДС индукции находят по формуле:

   

Примеры решения задач по теме «ЭДС индукции»

явление электромагнитной индукции, магнитный поток, закон электромагнитной индукции Фарадея, правило Ленца

9.5. Индукционный ток

9.5.1. Тепловое действие индукционного тока

Возникновение ЭДС приводит к появлению в проводящем контуре индукционного тока
, сила которого определяется по формуле

I
i
=
|

i
|
R
,

где ℰ
i

— ЭДС индукции, возникающая в контуре; R
— сопротивление контура.

При протекании индукционного тока в контуре выделяется теплота
, количество которой определяется одним из выражений:

Q
i
=
I
i
2
R
t
,
Q
i
=

i
2
t
R
,
Q
i
=
I
i
|

i
|
t
,

где I

i

— сила индукционного тока в контуре; R
— сопротивление контура; t
— время; ℰ
i

— ЭДС индукции, возникающая в контуре.

Мощность индукционного тока
вычисляется по одной из формул:

P
i
=
I
i
2
R
,
P
i
=

i
2
R
,
P
i
=
I
i
|

i
|
,

где I

i

— сила индукционного тока в контуре; R
— сопротивление контура; ℰ
i

— ЭДС индукции, возникающая в контуре.

При протекании индукционного тока в проводящем контуре через площадь поперечного сечения проводника переносится заряд
, величина которого вычисляется по формуле

q

i

= I

i

∆t
,

где I

i

— сила индукционного тока в контуре; Δt
— интервал времени, в течение которого по контуру течет индукционный ток.

Пример 21.
Кольцо, изготовленное из проволоки с удельным сопротивлением 50,0 ⋅ 10 −10 Ом ⋅ м, находится в однородном магнитном поле с индукцией 250 мТл. Длина проволоки равна 1,57 м, а площадь ее поперечного сечения составляет 0,100 мм 2 . Какой максимальный заряд пройдет по кольцу при выключении поля?

Решение
. Появление ЭДС индукции в кольце вызвано изменением потока вектора индукции, пронизывающего плоскость кольца, при выключении магнитного поля.

Поток индукции магнитного поля через площадь кольца определяется формулами:

  • до выключения магнитного поля

Ф 1 = B

1 S
 cos α,

где B

1 — первоначальное значение модуля индукции магнитного поля, B

1 = 250 мТл; S
— площадь кольца; α — угол между направлениями вектора магнитной индукции и вектора нормали (перпендикуляра) к плоскости кольца;

  • после выключения магнитного поля

Ф 2 = B

2 S
 cos α = 0,

где B

2 — значение модуля индукции после выключения магнитного поля, B

2 = 0.

∆Ф = Ф 2 − Ф 1 = −Ф 1 ,

или, с учетом явного вида Ф 1 ,

∆Ф = −B

1 S
 cos α.

Среднее значение ЭДС индукции, возникающей в кольце при выключении поля,

|

i
|
=
|
Δ
Ф
Δ
t
|
=
|

B
1
S
cos
α
Δ
t
|
=
B
1
S
|
cos
α
|
Δ
t
,

где ∆t
— интервал времени, за который происходит выключение поля.

Наличие ЭДС индукции приводит к появлению индукционного тока; сила индукционного тока определяется законом Ома:

I
i
=
|

i
|
R
=
B
1
S
|
cos
α
|
R
Δ
t
,

где R
— сопротивление кольца.

При протекании индукционного тока по кольцу переносится индукционный заряд

q
i
=
I
i
Δ
t
=
B
1
S
|
cos
α
|
R
.

Максимальному значению заряда соответствует максимальное значение функции косинус (cos α = 1):

q
i
max
=
I
i
Δ
t
=
B
1
S
R
.

Полученная формула определяет максимальное значение заряда, который пройдет по кольцу при выключении поля.

Однако для расчета заряда необходимо получить выражения, которые позволят найти площадь кольца и его сопротивление.

Площадь кольца — площадь круга радиусом r
, периметр которого определяется формулой длины окружности и совпадает с длиной проволоки, из которой изготовлено кольцо:

l
= 2πr
,

где l
— длина проволоки, l
= 1,57 м.

Отсюда следует, что радиус кольца определяется отношением

r
=
l
2
π
,

а его площадь —

S
=
π
r
2
=
π
l
2
4
π
2
=
l
2
4
π
.

Сопротивление кольца задается формулой

R
=
ρ
l
S
0
,

где ρ — удельное сопротивление материала проволоки, ρ = 50,0 × × 10 −10 Ом ⋅ м; S

0 — площадь поперечного сечения проволоки, S

0 = = 0,100 мм 2 .

Подставим полученные выражения для площади кольца и его сопротивления в формулу, определяющую искомый заряд:

q
i
max
=
B
1
l
2
S
0
4
π
ρ
l
=
B
1
l
S
0
4
π
ρ
.

Вычислим:

q
i
max
=
250

10

3

1,57

0,100

10

6
4

3,14

50,0

10

10
=
0,625
Кл
=
625
мКл.

При выключении поля по кольцу проходит заряд, равный 625 мКл.

Пример 22.
Контур площадью 2,0 м 2 и сопротивлением 15 мОм находится в однородном магнитном поле, индукция которого возрастает на 0,30 мТл в секунду. Найти максимально возможную мощность индукционного тока в контуре.

Решение
. Появление ЭДС индукции в контуре вызвано изменением потока вектора индукции, пронизывающего плоскость контура, при изменении индукции магнитного поля с течением времени.

Изменение потока вектора индукции магнитного поля определяется разностью

∆Ф = ∆BS
 cos α,

где ∆B
— изменение модуля индукции магнитного поля за выбранный интервал времени; S
— площадь, ограниченная контуром, S
= 2,0 м 2 ; α — угол между направлениями вектора магнитной индукции и вектора нормали (перпендикуляра) к плоскости контура.

Среднее значение ЭДС индукции, возникающей в контуре, при изменении индукции магнитного поля:

|

i
|
=
|
Δ
Ф
Δ
t
|
=
|
Δ
B
S
cos
α
Δ
t
|
=
Δ
B
S
|
cos
α
|
Δ
t
,

где ∆B
/∆t
— скорость изменения модуля вектора индукции магнитного поля с течением времени, ∆B
/∆t
= 0,30 мТл/с.

Появление ЭДС индукции приводит к появлению индукционного тока; сила индукционного тока определяется законом Ома:

I
i
=
|

i
|
R
=
Δ
B
S
|
cos
α
|
R
Δ
t
,

где R
— сопротивление контура.

Мощность индукционного тока

P
i
=
I
i
2
R
=

B
Δ
t)
2
S
2
R
cos
2
α
R
2
=

B
Δ
t)
2
S
2
cos
2
α
R
.

Максимальному значению мощности индукционного тока соответствует максимальное значение функции косинус (cos α = 1):

P
i
max
=

B
Δ
t)
2
S
2
R
.

Вычислим:

P
i
max
=
(0,30

10

3)
2
(2,0)
2
15

10

3
=
24

10

6
Вт
=
24
мкВт.

Максимальная мощность индукционного тока в данном контуре равна 24 мкВт.


Взаимосвязь электрических и магнитных полей замечена очень давно. Данную связь еще в 19 веке обнаружил английский ученый-физик Фарадей и дал ему название . Она появляется в тот момент, когда магнитный поток пронизывает поверхность замкнутого контура. После того как происходит изменение магнитного потока в течение определенного времени, в этом контуре наблюдается появление электрического тока.

Взаимосвязь электромагнитной индукции и магнитного потока

Суть магнитного потока отображается известной формулой: Ф = BS cos α. В ней Ф является магнитным потоком, S — поверхность контура (площадь), В — вектор магнитной индукции. Угол α образуется за счет направления вектора магнитной индукции и нормали к поверхности контура. Отсюда следует, что максимального порога магнитный поток достигнет при cos α = 1, а минимального — при cos α = 0.

Во втором варианте вектор В будет перпендикулярен к нормали. Получается, что линии потока не пересекают контур, а лишь скользят по его плоскости. Следовательно, определять характеристики будут линии вектора В, пересекающие поверхность контура. Для расчета в качестве единицы измерения используется вебер: 1 вб = 1в х 1с (вольт-секунда). Еще одной, более мелкой единицей измерения служит максвелл (мкс). Он составляет: 1 вб = 108 мкс, то есть 1 мкс = 10-8 вб.

Для исследования Фарадеем были использованы две проволочные спирали, изолированные между собой и размещенные на катушке из дерева. Одна из них соединялась с источником энергии, а другая — с гальванометром, предназначенным для регистрации малых токов. В тот момент, когда цепь первоначальной спирали замыкалась и размыкалась, в другой цепи стрелка измерительного устройства отклонялась.

Проведение исследований явления индукции

В первой серии опытов Майкл Фарадей вставлял намагниченный металлический брусок в катушку, подключенную к току, а затем вынимал его наружу (рис. 1, 2).

1 2

В случае помещения магнита в катушку, подключенную к измерительному прибору, в цепи начинает протекать индукционный ток. Если магнитный брусок удаляется из катушки, индукционный ток все равно появляется, но его направление становится уже противоположным. Следовательно, параметры индукционного тока будут изменены по направлению движения бруска и в зависимости от полюса, которым он помещается в катушку. На силу тока оказывает влияние быстрота перемещения магнита.

Во второй серии опытов подтверждается явление, при котором изменяющийся ток в одной катушке, вызывает индукционный ток в другой катушке (рис. 3, 4, 5). Это происходит в моменты замыкания и размыкания цепи. От того, замыкается или размыкается электрическая цепь, будет зависеть и направление тока. Кроме того, эти действия есть ни что иное, как способы изменения магнитного потока. При замыкании цепи он будет увеличиваться, а при размыкании — уменьшаться, одновременно пронизывая первую катушку.

3 4

5

В результате опытов было установлено, что возникновение электрического тока внутри замкнутого проводящего контура возможно лишь в том случае, когда они помещаются в переменное магнитное поле. При этом, поток может изменяться во времени любыми способами.

Электрический ток, появляющийся под действием электромагнитной индукции, получил название индукционного, хотя это и не будет током в общепринятом понимании. Когда замкнутый контур оказывается в магнитном поле, происходит генерация ЭДС с точным значением, а не тока, зависящего от разных сопротивлений.

Данное явление получило название ЭДС индукции, которую отражает формула: Еинд = — ∆Ф/∆t. Ее значение совпадает с быстротой изменений магнитного потока, пронизывающего поверхность замкнутого контура, взятого с отрицательным значением. Минус, присутствующий в данном выражении, является отражением правила Ленца.

Правило Ленца в отношении магнитного потока

Известное правило было выведено после проведения цикла исследований в 30-х годах 19 века. Оно сформулировано в следующем виде:

Направление индукционного тока, возбуждаемого в замкнутом контуре изменяющимся магнитным потоком, оказывает влияние на создаваемое им магнитное поле таким образом, что оно в свою очередь создает препятствие магнитному потоку, вызывающему появление индукционного тока.

Когда магнитный поток увеличивается, то есть становится Ф > 0, а ЭДС индукции снижается и становится Еинд

Если поток снижается, то наступает обратный процесс, когда Ф 0, то есть действие магнитного поля индукционного тока, происходит увеличение магнитного потока, проходящего через контур.

Физический смысл правила Ленца заключается в отражении закона сохранения энергии, когда при уменьшении одной величины, другая увеличивается, и, наоборот, при увеличении одной величины другая будет уменьшаться. Различные факторы влияют и на ЭДС индукции. При вводе в катушку поочередно сильного и слабого магнита, прибор соответственно будет показывать в первом случае более высокое, а во втором — более низкое значение. То же самое происходит, когда изменяется скорость движения магнита.

На представленном рисунке видно, как определяется направление индукционного тока с применением правила Ленца. Синий цвет соответствует силовым линиям магнитных полей индукционного тока и постоянного магнита. Они расположены в направлении полюсов от севера к югу, которые имеются в каждом магните.

Изменяющийся магнитный поток приводит к возникновению индукционного электрического тока, направление которого вызывает противодействие со стороны его магнитного поля, препятствующее изменениям магнитного потока. В связи с этим, силовые линии магнитного поля катушки направлены в сторону, противоположную силовым линиям постоянного магнита, поскольку его движение происходит в сторону этой катушки.

Для определения направления тока используется с правой резьбой. Он должен ввинчиваться таким образом, чтобы направление его поступательного движения совпадало с направлением индукционных линий катушки. В этом случае направления индукционного тока и вращения рукоятки буравчика будут совпадать.

На рисунке показано направление индукционного тока,возникающего в короткозамкнутой проволочной катушке,когда относительно нее перемещают

магнит.Отметьте,какие из следующих утверждений правильные,а какие- неправильные.
А.Магнит и катушка притягиваются друг к другу.
Б. Внутри катушки магнитное поле индукционного тока направленно вверх.
В. Внутри катушки линии магнитной индукции поля магнита направлены вверх.
Г. Магнит удаляют от катушки.

1. Первый закон Ньютона?

2. Какие системы отсчета являются инерциальными и неинерциальными? Приведите примеры.
3. В чем состоит свойство тел, называемое инертностью? Какой величиной характеризуется инертность?
4. Какова связь между массами тел и модулями ускорений, которые они получают при взаимодействии?
5. Что такое сила и чем она характеризуется?
6. Формулировка 2 закона Ньютона? Какова его математическая запись?
7. Как формулируется 2 закон Ньютона в импульсной форме? Его математическая запись?
8. Что такое 1 Ньютон?
9. Как движется тело, если к нему приложена сила постоянная по модулю и направлению? Как направлено ускорение, вызванное действующей на него силой?
10. Как определяется равнодействующая сил?
11. Как формулируется и записывается 3 закон Ньютона?
12. Как направлены ускорения, взаимодействующих между собой тел?
13. Приведите примеры проявления 3 закона Ньютона.
14. Каковы границы применимости всех законов Ньютона?
15. Почему мы можем считать Землю инерциальной системой отсчета, если она двигается с центростремительным ускорением?
16. Что такое деформация, какие виды деформации вы знаете?
17. Какая сила называется силой упругости? Какова природа этой силы?
18. Каковы особенности силы упругости?
19. Как направлена сила упругости (сила реакции опоры, сила натяжения нити?)
20. Как формулируется и записывается закон Гука? Каковы его границы применимости? Постройте график, иллюстрирующий закон Гука.
21. Как формулируется и записывается закон Всемирного тяготения, когда он применим?
22. Опишите опыты, по определению значения гравитационной постоянной?
23. Чему равна гравитационная постоянная, каков ее физический смысл?
24. Зависит ли работа силы тяготения от формы траектории? Чему равна работа силы тяжести по замкнутому контуру?
25. Зависит ли работа силы упругости от формы траектории?
26. Что вы знаете о силе тяжести?
27. Как вычисляется ускорение свободного падения на Земле и других планетах?
28. Что такое первая космическая скорость? Как ее вычисляют?
29. Что называют свободным падением? Зависит ли ускорение свободного падения от массы тела?
30. Опишите опыт Галилео Галилея, доказывающий, что все тела в вакууме падают с одинаковым ускорением.
31. Какая сила называется силой трения? Виды сил трения?
32. Как вычисляют силу трения скольжения и качения?
33. Когда возникает сила трения покоя? Чему она равна?
34. Зависит ли сила трения скольжения от площади соприкасающихся поверхностей?
35. От каких параметров зависит сила трения скольжения?
36. От чего зависит сила сопротивления движению тела в жидкостях и газах?
37. Что называют весом тела? В чем заключается различие между весом тела и силой тяжести, действующей на тело?
38. В каком случае вес тела численно равен модулю силы тяжести?
39. Что такое невесомость? Что такое перегрузка?
40. Как вычислить вес тела при его ускоренном движении? Изменяется ли вес тела, если оно движется по неподвижной горизонтальной плоскости с ускорением?
41. как изменяется вес тела при его движении по выпуклой и вогнутой части окружности?
42. Каков алгоритм решения задач при движении тела под действием нескольких сил?
43. Какая сила называется Силой Архимеда или выталкивающей силой? От каких параметров зависит эта сила?
44. По каким формулам можно вычислить силу Архимеда?
45. При каких условиях тело, находящееся в жидкости плавает, тонет, всплывает?
46. Как зависит глубина погружения в жидкость плавающего тела от его плотности?
47. Почему воздушные шары наполняют водородом, гелием или горячим воздухом?
48. Объясните влияние вращения Земли вокруг своей оси на значение ускорения свободного падения.
49. Как изменяется значение силы тяжести при: а) удалении тела от поверхности Земли, Б) при движении тела вдоль меридиана, параллели

электрической цепи?

3. Каков физический смысл ЭДС? Дать определение вольту.

4. Соединить на короткое время вольтметри источником электрической энергии, соблюдая полярность. Сравнить его показания с вычислением по результатам опыта.

5. От чего зависит напряжение на зажимах источников тока?

6. Пользуясь результатами измерений, определить напряжение на внешней цепи (если работа выполнена I методом), сопротивление внешней цепи (если работа выполнена II методом).

6 вопрос во вложение вычисление

Помогите пожалуйста!

1. При каких условиях появляются силы трения?
2. От чего зависят модуль и направление силы трения покоя?
3. В каких пределах может изменяться сила трения покоя?
4. Какая сила сообщает ускорение автомобилю или тепловозу?
5. Может ли сила трения скольжения увеличить скорость тела?
6. В чем состоит главное отличие силы сопротивления в жидкостях и газах от силы трения между двумя твердыми телами?
7. Приведите примеры полезного и вредного действия сил трения всех видо

Как мы уже выяснили, электрический ток способен порождать магнитные поля.Свитый в катушку проводник замыкается на гальванометре (рис. 3.19). Если вдвигать в катушку постоянный магнит, то гальванометр покажет наличие тока в течение всего промежутка времени, пока магнит перемещается относительно катушки. При выдергивании магнита из катушки гальванометр показывает наличие тока противоположного направления. Изменения направления тока происходит при изменении вдвигаемого или выдвигаемого полюса магнита.

Аналогичные результаты наблюдались при замене постоянного магнита электромагнитом (катушкой с током). Если обе катушки закрепить неподвижно, но в одной из них менять значение тока, то в этот момент в другой катушке наблюдается индукционный ток.

ЯВЛЕНИЕ ЭЛЕКТРОМАГНИТНОЙ ИНДУКЦИИ состоит в возникновении электродвижущей силы (э.д.с.) индукции в проводящем контуре, через который меняется поток вектора магнитной индукции. Если контур является замкнутым, то в нем возникает индукционный ток.

Открытие явления электромагнитной индукции:

1) показало взаимосвязь между электрическим и магнитным полем
;

2) предложило способ получения электрического тока
с помощью магнитного поля.

Основные свойства индукционного тока
:

1. Индукционный ток возникает всегда, когда происходит изменение сцепленного с контуром потока магнитной индукции.

2. Сила индукционного тока не зависит от способа изменения потока магнитной индукции, а определяется лишь скоростью его изменения.

Опытами Фарадея было установлено, что величина электродвижущей силы индукции пропорциональна скорости изменения магнитного потока, пронизывающего контур проводника (закон электромагнитной индукции Фарадея)

Или , (3.46)

где (dF) – изменение потока в течении времени (dt).МАГНИТНЫМ ПОТОКОМ
или ПОТОКОМ МАГНИТНОЙ ИНДУКЦИИ
называется величина, которая определяется на основе следующего соотношения: (магнитный поток через поверхность площадью S
): Ф=ВScosα, (3.45), угол a – угол между нормалью к рассматриваемой поверхности и направлением вектора индукции магнитного поля

единица магнитного потока
в системе СИ носит название вебер
– [Вб=Тл×м 2 ].

Знак «–» в формуле означает, что э.д.с. индукции вызывает индукционный ток, магнитное поле которого противодействует всякому изменению магнитного потока, т.е. при >0 э.д.с. индукции e И

э.д.с. индукции измеряется в вольтах

Для нахождения направления индукционного тока существует правило Ленца (правило установлено в 1833 г.): индукционный ток имеет такое направление, что создаваемое им магнитное поле стремится компенсировать изменение магнитного потока, вызвавшее этот индукционный ток.

Например, если вдвигать северный полюс магнита в катушку, т. е. увеличивать магнитный поток через его витки, в катушке возникает индукционный ток такого направления, что на ближайшем к магниту конце катушки возникает северный полюс (рис.3.20). Итак, магнитное поле индукционного тока стремится нейтрализовать вызвавшее его изменение магнитного потока.

Не только переменное магнитное поле порождает индукционный ток в замкнутом проводнике, но и при движении замкнутого проводника длиной l в постоянном магнитном поле (В) со скоростью v в проводнике возникает эдс:

a (B Ùv) (3.47)

Как вы уже знаете, электродвижущая сила
в цепи– это результат действия сторонних сил. При движении проводника в магнитном поле роль сторонних сил
выполняет сила Лоренца
(которая действует со стороны магнитного поля на движущийся электрический заряд). Под действием этой силы происходит разделение зарядов и на концах проводника возникает разность потенциалов. Э.д.с. индукции в проводнике является работой по перемещению единичных зарядов вдоль проводника.

Направление индукционного тока
можно определитьпо правилу правой руки:
Вектор В входит в ладонь, отведенный большой палец совпадает с направлением скорости проводника, а 4 пальца укажут направление индукционного тока.

Таким образом переменное магнитное поле вызывает появление индуцированного электрического поля. Оно не потенциально
(в отличие от электростатического), т.к. работа
по перемещению единичного положительного заряда равна э.д.с. индукции
, а не нулю.

Такие поля называются вихревыми. Силовые линии вихревого
электрического поля – замкнуты сами на себя,
в отличие от линий напряженности электростатического поля.

Э.д.с. индукции возникает не только в соседних проводниках, но и в самом проводнике при изменении магнитного поля тока, идущего по проводнику. Возникновение э.д.с. в каком-либо проводнике при изменении в нем самом силы тока (следовательно, магнитного потока в проводнике) называется самоиндукцией, а ток, индуцируемый в этом проводнике, – током самоиндукции.

Ток в замкнутом контуре создает в окружающем пространстве магнитное поле, напряженность которого пропорциональна силе тока I. Поэтому магнитный поток Ф, пронизывающий контур, пропорционален силе тока в контуре

Ф=L×I, (3.48).

L – коэффициент пропорциональности, который носит название коэффициента самоиндукции, или, просто, индуктивности. Индуктивность зависит от размеров и формы контура, а также от магнитной проницаемости среды, окружающей контур.

В этом смысле индуктивность контура — аналог
электрической емкости уединенного проводника, которая также зависит только от формы проводника, его размеров и диэлектрической проницаемости среды.

Единица индуктивности — генри (Гн)
: 1Гн — индуктивность такого контура, магнитный поток самоиндукции которого при токе в 1А равен 1Вб (1Гн=1Вб/А=1В·с/А).

Если L=const, то э.д.с. самоиндукции можно представить в следующем виде:

, или , (3.49)

где DI (dI) – изменение тока в цепи, содержащей катушку индуктивности (или контур) L, за время Dt (dt). Знак «–» в этом выражении означает, что э.д.с. самоиндукции препятствует изменению тока (т. е. если ток в замкнутом контуре уменьшается, то э.д.с. самоиндукции приводит к возникновению тока того же направления и наоборот).

Одним из проявлений электромагнитной индукции является возникновение замкнутых индукционных токов в сплошных проводящих средах: металлических телах, растворах электролитов, биологических органах и т.д. Такие токи носят название вихревых токов или токов Фуко. Эти токи возникают при перемещении проводящего тела в магнитном поле и/или при изменении со временем индукции поля, в которое помещены тела.
Сила токов Фуко зависит от электрического сопротивления тел, а также от скорости изменения магнитного поля.

Токи Фуко также подчиняются правилу Ленца

: их магнитное поле направлено так, чтобы противодействовать изменению магнитного потока, индуцирующему вихревые токи.

Поэтому массивные проводники тормозятся в магнитном поле. В электрических машинах, для того чтобы минимизировать влияние токов Фуко, сердечники трансформаторов и магнитные цепи электрических машин собирают из тонких пластин, изолированных друг от друга специальным лаком или окалиной.

Вихревые токи вызывают сильное нагревание проводников. Джоулево тепло, выделяемое токами Фуко
, используется в индукционных металлургических печах
для плавки металлов, согласно закону Джоуля-Ленца
.

«Магнетизм». Тестирование по физике для подготовки учащихся к ЕНТ

Тема № 10: «Магнетизм».

Тестирование по физике для подготовки учащихся к ЕНТ.

Автор: Учитель высшей категории Плотникова Г.Д.

25.03.2016 г.

Магнетизм.

Магнитное поле – особая форма материи

МП возникает вокруг:

1. Движущихся зарядов (электрического тока)

2. Постоянных магнитов.

МП оказывает ориентирующее действие на:

1. Рамку с током (проводник с током)

2. Магнитную стрелку

1.

Вектор магнитной индукции для проводника с током.

В=F/Iℓ

Тл=Н/А·м

В-вектор магнитной индукции, Тл=Н/Ам:

F=ВIℓ; I=F/Вℓ;

2.

Вектор магнитной индукции для рамки с током.

В=М/IS

М- момент вращающий рамку с током, Н·м.

S- площадь рамки с током, м2.

3.

Момент сил.

М= F·d

F-сила, Н

d-плечо силы, м.

4.

Максимальный вращательный момент, действующий на рамку с током в МП.

М= BIS

Н·м= (Н/А·м)·А·м2

I-сила тока в рамке, А.

S-площадь рамки, м2.

Направление В определяется (по определению и индикаторам)

5.

По магнитным линиям- линии касательные к которым в каждой точке совпадают с В.

По рамке с током:

По правилу «правой руки».

По магнитной стрелке:

В- направлено от южного к северному полюсу магнитной стрелки свободно устанавливающейся в МП.

6.

Сила Ампера (закон Ампера) сила, действую-щая на заряженную частицу в МП.

FА=ВIℓsinα

α-угол между векторами В и I,

7.

Направление силы Ампера .

«Правило левой руки».

В — перпендикулярно в ладонь!

υ — четыре пальца по направлению тока!

Fлнаправление большого пальца!

8.

Сила Лоренца-сила, действующая на заря-женную частицу в МП.

Fл= FА/N

Fл=q·В·υ·sinα

N-число заряженных частиц,

q-заряд частицы, Кл.

υ-скорость частицы, м/с.

Α- угол между В и υ

9.

Направление силы Лоренца.

«Правило левой руки».

В — перпендикулярно в ладонь!

υ — четыре пальца «+» частицы!

Fл-большой палец!

Движение заряженной частицы в магнитном поле, если она влетает:

10.

а) вдоль линий напряженности:

МП не влияет на движение

Траектория-прямая, движение равномерное.

11.

б) перпендикулярно линиям напряженности

Условие движения:

FЛ=mац

qυВ = mυ2/R

υ=соnst

Т=2π R/υ=π·m/q·1/В

Траектория-окружность, движение равноускоренное.(центростремительное).

R= mυ/qВ

υ=(q/m)·В R

12.

в) под углом α к линиям напряженности

υ=υ·cos α

R= m·υ·sinα/q·В

Т=2πR/υ=

2πm/qВ- h=υ·Т=

=υ·cos α·2π·m/q·В

Траектория-по винтовой линии,

движение по ОХ-равномерное,

по ОУ-равноускоренное

υ-скорость поступательного движения частицы вдоль силовой линии.

R -радиус винтовой линии.

Т- период (время одного оборота)

h -шаг винтовой линии

13.

Магнитный поток

Ф=ВScos α

Вб=Тл·м2

В-вектор магнитной индукции, Тл=Н/Ам:

S-площадь рамки, м2

α -угол между В и нормалью n к контуру.

14.

Закон электромагнитной индукции (закон Фарадея).

ε =ΔΦ/Δt

В=Вб/с

ΔΦ21-изменение магнитного потока, Вб.

ΔΦ/Δt-скорость изменения магнитного потока, Вб/с.

15.

Закон электромагнитной индукции для нескольких витков.

ε = — N·ΔΦ/Δt

N— число витков контура.

16.

Закон электромагнитной индукции для неподвижного контура в котором меняется МП

εинд = — N В/Δt

В ,м2Тл/с =

м2 (Н/А·м) /с = Дж/А·с = ВсА/Ас=В

ΔВ/Δt-скорость изменения МП, Тл/с.

17.

ЭДС индукции в движущихся проводниках

ε=В·ℓ·υ·sinα

В-вектор магнитной индукции, Тл=Н/Ам:

длина проводника, м; υ-скорость проводника, м/с.

18.

Индукционный ток, закон Ома.

I=ε/R

А=В/Ом

19.

Заряд q, проходящий через контур при изменении магнитного потока

q = ΔΦ/R

Вб=Кл·Ом=А·с·Ом=В·с

ΔΦ21-изменение магнитного потока.

R- сопротивление контура, Ом.

20.

Магнитный поток, возникающий в контуре, создаваемый собственным током контура.

Ф=LI

Вб=Гн·А

L- коэффициент самоиндукции (индуктивность) контура, Гн.

21.

Эдс самоиндукции, возника-ющая при изменении тока.

ε = — L·ΔI/Δt

ΔI/Δt- скорость изменения силы тока, А/с.

22.

Коэффициент самоиндукции (индуктивность) контура

L = ε· Δt /ΔI

Гн=Вс/А

23.

Энергия магнитного поля

W=LI2/2

Дж=Гн·А2=( Вс/А)А2=В·с·А

24.

Магнитная проницаемость вещества.

μ=В/В0

В-вектор магнитной индукции в среде, Тл.

В0— вектор магнитной индукции в вакууме.

Магнетики-это вещества, которые во внешнем МП намагничиваются, т.е. имеют свое МП.

25.

Диамагнетики

Парамагнетики

Ферромагнетики

μ

μ1

μ1

серебро, цинк, азот, свинец, кварц, висмут

платина, никель жидкий кислород

железо, сталь, чугун, никель, некоторые сплавы

26.

Температура Кюри

Тктабличное значение

Температура, при которой исчезают магнитные свойства вещества.

Устные вопросы:

1. Магнитное поле. Магнитное поле постоянных магнитов. Магнитное поле Земли. Магнитные линии.

2. Магнитное взаимодействие. Опыты Эрстеда. Магнитное поле (однородное, неоднородное). Магнитные линии.

3. Магнитная индукция(величина и направление).

4. Магнитное поле прямого и кругового тока, соленоида.

5. Закон Ампера. Направление силы Ампера.

6. Сила Лоренца. Направление силы Лоренца. Движение частицы в магнитном поле.

7. Закон электромагнитной индукции. Самоиндукция. Индуктивность

8. Магнитные свойства вещества (диа-, пара-, ферромагнетики), гипотеза Ампера.

1. МП и магнитное взаимодействие.

1. Если силовые линии замкнуты, то поле

A) электрическое. B) гравитационное. C) электрическое и магнитное. D) вихревое. E) электромагнитное.

2. Два параллельных проводника, по которым токи текут в противоположных направлениях

A) не взаимодействуют. B) притягиваются. C) подвергаются действию силы Кулона. D) отталкиваются. E) вращаются в одну сторону.

3.Если смотреть сверху, то линии магнитной индукции направлены

A) по часовой стрелке. B) против направлению тока. C) против часовой стрелки. D) по направлению тока. E) по радиусу от проводника.

4. Движущийся электрический заряд создает.

A) только электрическое поле. B) только магнитное поле. C) и электрическое и магнитное поле. D) либо электрическое, либо магнитное, в зависимости от скорости. E) электрическое или магнитное в зависимости от среды.

5. Мимо сидящего ученика учитель проносит заряженный шарик. В этом случае

A) Учитель и ученик обнаружат только электрическое поле. B) Учитель и ученик могут обнаружить оба поля. C) Ученик обнаружит магнитное поле, учитель- электрическое. D) Ученик обнаружит и электрическое и магнитное поля, учитель только электрическое. E) Ученик обнаружит электрическое поле, учитель- магнитное.

6. Если силы взаимодействия направлены так, как показано на рисунке, то

A) токи по первому и второму проводникам идут вверх. B) токи по первому и второму проводникам идут вниз. C) тока в проводниках нет. D) токи идут по противоположным направлениях. E) нет тока в одном из проводников.

7. Длинная катушка в виде пружины очень малой жесткости находится на гладком столе. Если по катушке пропустить электрический ток, то

A) катушка сожмется. B) катушка подпрыгнет. C) ничего не изменится.

D) катушка будет вращаться. E) катушка растянется.

8. Линия индукции магнитного поля – это

A) производная линия в пространстве магнитного поля. B) линия, касательная к которой в каждой точке совпадает с направлением вектора индукции в этой точке. C) линия, соединяющая северный и южный полюсы магнита. D) линия, идущая от точки с большим значением индукции к точке с меньшим значением индукции. E) линия, соединяющая точки магнитного поля с одинаковыми по величине значения индукции

9. Магнитное поле создают

А) магнитные заряды. В) статические электрические заряды. С) заряженные проволочные катушки. D) движущиеся электрические заряды. Е) длинные заряженные проводники.

10. При пропускании постоянного тока через проводник вокруг него возникает магнитное поле. Оно обнаруживается по расположению стальных опилок на листе бумаги, или по повороту магнитной стрелки. В каком случае это магнитное поле исчезнет?

А) если выключить электрический ток в проводе. В) если убрать стальные опилки. С) если убрать магнитную стрелку. Д) если убрать стальные опилки и магнитную стрелку. Е) однажды созданное магнитное поле ни когда не исчезнет.

11. Где находятся магнитные полюса Земли?

А) на северном географическом полюсе Земли. В) на южном географическом полюсе Земли. С) северный магнитный полюс находится на северном географическом полюсе Земли. Д) северный магнитный полюс находится на южном географическом полюсе Земли. Е) северный и южный магнитные полюса находится соответственно на южном и северном географическом полюсе Земли.

12. Можно ли разделить полосовой магнит на две части?

А) нельзя, магнитное поле исчезнет. В) можно, при этом один магнит будет северный, а другой южный. С) можно, при этом оба магнита будут иметь по два полюса: северный и южный. Д) нельзя, оба магнита будут одного северного полюса. Е) нельзя, оба магнита будут одного южного полюса.

2. Вектор магнитной индукции магнитного поля.

1. На проводник длиной 40 см, расположенный под углом 300 к линиям индукции магнитного поля, действует сила 0,4 Н. Если в проводнике сила тока равна 2 А, то индукция магнитного поля равна

A)1 Тл. B) 3 Тл. C) √3/2 Тл. D) 2 Тл. E) 4 Тл.

2. По двум параллельным бесконечно длинным прямолинейным проводникам С и Е текут одинаковые по величине и направленные от нас токи. Если АС=СD=DЕ=ЕF=FG, то индукция результирующего магнитного поля равна нулю

A) в точке А. B) во всех точках. C) в точкеD. D) в точке G. E) в точке F.

3. Если магнитные поля однородные, то линии магнитной индукции всегда

A) совпадают с направлением тока в проводнике. B) направлены против тока в проводнике. C) направлены перпендикулярно проводнику. D) направлены против вектора В. E) совпадают с направлением вектора В.

4. Силовой характеристикой магнитного поля является

A) Магнитный поток. B) Сила Ампера. C) Сила тока в проводнике. D) Вектор магнитной индукции. E) Индуктивность.

3. Силы Ампера.

1. Магнитное поле на рамку с током оказывает

A) световое действие. B) химическое действие. C) ионизирующее действие.

D) механическое действие. E) тепловое действие.

2. По какому правилу можно определить направление силовых линий магнитного поля проводника с током?

А) ленца. В) левой руки. С) правой руки. Д) параллелограмма. Е) буравчика.

3. По проводнику пропустили постоянный ток. Магнитное поле этого тока можно обнаружить с помощью:

А) электростатика. В) амперметра. С) вольтметра. Д) стальных опилок.

Е) деревянных опилок.

Решите задачи:

2. Однородное магнитное поле с индукцией 10 мТл действует на проводник длиной 10 см с током 50 А. Если проводник перпендикулярен линиям магнитной индукции, то сила этого действия

A) 500 Н. B) 5·103Н. C) 5·10-2Н. D) 50 Н. E) 5 Н.

3. На прямолинейный проводник длиной 20 см, расположенный перпендикулярно направлению магнитного поля, действует сила 8 Н. Если сила тока в проводнике равна 40 А, то магнитная индукция поля

A) 5 Тл. B)1 Тл. C) 3 Тл. D) 4 Тл. E) 2 Тл.

4. Проводник длиной 20 см и массой 4 г, расположен горизонтально, перпендикулярно линиям индукции магнитного поля. По нему течет ток 10 А. Сила тяжести, действующая на проводник, уравновесилась силой Ампера. Индукция магнитного поля равна

A) 20 мТл. B) 12 мТл. C) 30 мТл. D) 18 мТл. E) 15 мТл

5. *Проводник длиной 60 см с током 10 А может располагаться под разными углами к линиям магнитной индукций. Если магнитная индукция поля равна

1,5 Тл, то наибольшее и наименьшее значение силы Ампера соответственно равны

A) 12 Н, 1 Н. B) 8 Н, 0. C) 16 Н, 2 Н. D) 7 Н, 0. E) 9 Н, 0.

6. *Проводник длиной ℓ и массой m1 подвешенный горизонтально на двух тонких нитях, находится в магнитном поле с индукцией, направленной вертикально вниз. При пропускании по проводнику тока I, нити отклонились от вертикали на угол φ. Индукция магнитного поля равна

A) В =Iℓ/mg·sin φ. B) В = FA·Iℓsinφ. C) В = FA·sinφ/mg. D) В = mg·tgφ/Iℓ.

E) В = FA/FT·cosφ.

7. *Проводник длиной 50 см и массой 20 г, подвешенный на двух тонких нитях, помещен в магнитное поле с индукцией 0,4 Тл, направленной горизонтально. Натяжение нитей исчезает при силе тока, равной (g=10 м/с2)

A) 4 А. B) 1 А. C) 5 А. D) 2 А. E) 0,5А.

4. Силы Лоренца

1. Силу Лоренца можно определить по формуле

A). B). C) F=pgV. D) F=k . E) F=.

2. Для определения направления силы Лоренца используется

A) правило буравчика. B) принцип суперпозиции. C) закон сохранения электрического заряда. D) правило левой руки. E) закон сохранения энергии.

3. Электрон движется горизонтально в направлении «Юг-Север» и влетает в вертикальное однородное магнитное поле, направленное вверх. При этом сила Лоренца

A) Отклоняет электрон на запад. B) На электрон не действует. C) Отклоняет электрон вниз. D) Отклоняет электрон вверх. E) Отклоняет электрон на восток.

4. На рисунке показано однородное постоянное во времени магнитное поле. Электрон, помещенный в это поле и не имеющий начальной скорости

A) движется равноускоренно вправо. B) остается неподвижным. C) движется по окружности по часовой стрелке. D) движется по окружности против часовой стрелки. E) движется равноускоренно вправо.

5. Заряженная частица с зарядом q движется перпендикулярно однородному магнитному полю со скоростью υ. Если заряд частицы увеличить в 2 раза при сохранении всех других параметров, то радиус траектории движения частицы

A) увеличится в 2 раза. B) не изменится. C) увеличится в 4 раза. D) уменьшится в 4 раза. E) уменьшится в 2 раза.

Решите задачи:

6. В магнитном поле с индукцией 2 Тл движется электрический заряд Кл со скоростью 4 м/c. Если вектор скорости υ движения заряда перпендикулярен вектору индукции магнитного поля, то сила, действующая на заряд со стороны магнитного поля, равна

A) 0·Н. B) 0,5·Н. C) 2·Н. D) Н. E) 6·Н.

7. Частица с электрическим зарядом 8·10-19 Кл движется со скоростью 500 км/с в магнитном поле с индукцией 5 Тл. Угол между векторами скорости и индукции 30˚. Сила Лоренца равна (sin30˚ = 0,5)

A) 10-15Н. B) 2·10-14 Н. C) 2·10-12 Н. D) 4·10-12Н. E) 10-12Н.

8. *В направлении, перпендикулярном линиям магнитной индукции, в магнитное поле влетает электрон со скоростью 107 м/с. Если известно, что электрон описал окружность радиусом 10 мм. То индукция магнитного поля равна (q=-1.6·10-19Кл; me=9.1·10-31 кг)

A) ≈6,5 мТл. B) ≈7,5 мТл. C) ≈15 мТл. D) ≈5,7мТл. E) ≈1,5 мТл.

9.*Протон движется в магнитном поле с индукцией 0,5 Тл. Частота обращения протона равна (mр=1,67∙10-27кг; e=1,6∙10-19 Кл)

A) 7,6 ∙106 с-1. B) 3,4∙10-6 с-1. C) 2,8∙105 с-1. D) 4,3∙108 с-1. E) 1,9∙107 с-1.

10. *Электрон начинает двигаться в электрическом поле из стояния покоя и, пройдя разность потенциалов 220 В, попадает в однородное магнитное поле с индукцией 5 мТл, где он движется по круговой траектории радиусом 1,01 м. Если заряд электрона -1,6·10-19 Кл, то его масса

A) ≈15,1·10-31кг. B) ≈9,1·10-29 кг. C) ≈9,1·10-31 кг. D) ≈19,1·10-31 кг. E) ≈9,1·10-30 кг.

11. *В однородном магнитном поле с индукцией 5∙102 Тл находится прямоугольная рамка с площадью 0,03 м2. Вначале плоскость рамки составляла угол 30º с вектором индукции магнитного поля, затем рамку повернули так, что вектор индукции стал параллелен плоскости рамки. При этом через рамку прошёл заряд

3 мкКл. Сопротивление рамки равно (sin30º=0,5)

A) 0,12 Ом. B) 0,025 Ом. C) 0,125 Ом. D) 0,0025 Ом. E)25 Ом.

12. *В однородном магнитном поле с индукцией 5·10-3 Тл, находится прямоугольная рамка. Сопротивление рамки равно 0,5 Ом. Вначале плоскость рамки составляла угол 30˚ с вектором индукции магнитного поля. Затем рамку повернули так, что вектор индукции стал параллелен плоскости рамки, при этом через рамку прошел заряд 5 мкКл. Площадь рамки равна (sin30˚ = 0,5)

A) 8 см2. B) 20 см2. C)10 см2. D) 12 см2. E) 15 см2.

5. Движение заряженной частицы в магнитном поле.

1. Если начальная скорость заряженной частицы перпендикулярна линиям магнитной индукции, то в однородном магнитном поле она.

A)движется по окружности. B) движется по параболе. C) находится в покое.

D) движется по спирали. E) движется по прямой.

2. Заряженнаячастица, влетевшая в однородное магнитное поле вдоль линий магнитной индукции В, движется

A) против вектора магнитной индукции В. B) прямолинейно. C) по окружности.

D) по спирали. E) по параболе.

3. Заряженная частица массой m движется перпендикулярно однородному магнитному полю со скоростью υ. Если массу частицы увеличить в 2 раза, то радиус траектории движения частицы.

A) не изменится. B) уменьшится в 2 раза. C) увеличится в √2 раза.

D) уменьшится в √2 раза. E) увеличится в 2 раза.

4. Заряженная частица движется перпендикулярно однородному магнитному полю. При увеличении скорости заряда в 2 раза и увеличении индукции магнитного поля в 2 раза, сила, действующая на электрический заряд со стороны магнитного поля.

A) уменьшится в 2 раза. B) увеличится в 4 раза. C) не изменится.

D) уменьшится в 4 раза. E) увеличится в 2 раза.

5. Если начальная скорость заряженной частицы перпендикулярна линиям магнитной индукции, то в однородном магнитном поле она

A) движется по окружности. B) движется по параболе. C) находится в покое.

D) движется по спирали. E) движется по прямой.

*6. Заряженная частица движется перпендикулярно силовым линиям однородного магнитного поля со скорость . Если скорость частицы увеличить в 2 раза, то период обращения

A) увеличится в 2 раза. B) уменьшится в 4 раза. C) увеличится в 4 раза. D) не изменится. E) уменьшится в 2 раза.

Решите задачи:

7.α-частица, влетевшая в магнитное поле со скоростью 10м/с, движется по траектории с радиусом кривизны 1 ,038 м. Индукция магнитного поля равна

(mα частицы=6,646 ∙10 кг; qα частицы =3,2∙10Кл)

A) 2,5мТл. B) 10 мТл. C) 5 мТл. D)20 мТл. E) 0,4 мТл.

*8. Протон, прошедший ускоряющую разность потенциалов 600 В, влетает в однородное магнитное поле с магнитной индукцией 0,3 Кл и движется по окружности. Если заряд протона 1,6·10-19 Кл, масса протона 1,67·10-27кг, то радиус окружности

A) 0,12 м. B)0,012 м. C) 1,2 мм. D) 1,2 м. E) 120 м.

*9. Электрон начинает двигаться в электрическом поле из состояния покоя и пройдя разность потенциалов 220 В попадает в однородное магнитное поле с индукцией

5 мТл где он движется по круговой траектории радиусом 0,01м. Если заряд электрона -1.6·10 -19Кл, то его масса

A) ≈9,1 10-31кг. B) ≈9,1 10-29кг. C) ≈9,1 10-30кг. D) ≈1,19 10-31кг. E) ≈15,1 10-31кг.

*10. При резком торможении электрона возникает рентгеновские длиной волны

0,6 нм. Если вся кинетическая энергия электрона превратилась в энергию электромагнитного поля, то максимальная скорость электрона равна (релятивистским эффектом пренебречь, me=9,1·10-31 кг, c=3·108 м/c,

h=6,62·10-34Дж·с)

A) ≈3,7·107 м/c. B) ≈2,7·107м/c. C) ≈4,7·106 м/c. D) ≈2,7·106м/c. E) ≈1,7·107 м/c.

*11.Электрон движется в вакууме в однородном магнитном поле в индукцией 5 мТл, его скорость равна 107 м/с и направлена перпендикулярно по линиям магнитной индукции. Сила Лоренца и радиус дуги окружности, по которой движется электрон, соответственно равны (qe=1/6·10-19Кл; me=9,1·10-31кг)

А) 18·10 -15 Н, ≈1·10-2 м. В) 8·10-15 H, ≈2·10-2 м. С) 8·10-15 H, ≈1·10-2 м.

D) 6·10-15 H, ≈2,1·10-2 м. E) 8·10-15 H, ≈4·10-2 м.

*12. В направлении, перпендикулярном линиям магнитной индукции, в магнитном поле влетает электрон со скоростью 107 м/с. Если известно, сто электрон описал окружность радиусом 10 мм, то индукция магнитного поля равна (qe= -1,6·10-19Кл; me=9,1 10-31 кг)

A) ≈6,5 мТл. B) ≈7,5 мТл. C) ≈5,7 мТл. D) ≈1,5 мТл. E) ≈15 мТл.

6. Магнитные свойства.

1. Температурой Кюри называют

A) температуру, при которой исчезают различия в физических свойствах жидкости и ее насыщенного пара. B) температуру, при которой вещество становиться сверхпроводником. C) температуру, при которой жидкость кипит. D) температуру, при которой исчезают ферромагнитные свойства вещества. E) температуру, при которой вещество плавиться.

2. Магнитные свойства вещества характеризует.

A) вектор магнитной индукции. B) магнитная проницаемость среды. C) магнитная постоянная. D) Вектор напряженности. E) Плотность вещества.

3. Безразмерной физической величиной в системе единиц СИ является

A) поток магнитной индукции. B) сила тока. C) магнитная постоянная.

D) магнитная индукция. E) магнитная проницаемость среды.

4. Четыре одинаковые катушки включены последовательны в электрическую цепь постоянного тока. Катушка 1- без сердечника, в катушке 2- железный, в катушке 3- алюминиевый, в катушке 4- медный сердечник. (Алюминий – парамагнетик, медь – диамагнетик, железо – ферромагнетик). Наименьший магнитный поток создает катушка.

A) 4. B) 2. C) 3. D) потоки одинаковы. E) 1.

5. Величина индукции магнитного поля внутри катушки с током при введении в него железного сердечника.

A) уменьшается незначительно. B) уменьшится. C) увеличится незначительно. D) не изменится. E) увеличится значительно.

6. Для магнитной проницаемости ферромагнетиков справедливо соотношение

A)1.B) 1. C) D) E) =1.

7. Если магнитная индукция в чугуне равна 0,4 Тл, а его магнитная проницаемость 500, то индукция внешнего магнитного поля равна

A) 0,08 Тл. B) 0,8 мТл. C) 0,8 мкТл. D) 0,06 Тл. E) 0,8 Тл.

8. Если магнитная индукция в чугуне равна 0.4 Тл, а индукция внешнего магнитного поля равна 0,8 мТл, то магнитная проницаемость чугуна

A) 1000. B) 2000. C) 1500. D) 500. E) 750.

9. Когда металлический брусок поместили во внешнее магнитное поле с индукцией 0,4 мТл, его магнитная проницаемость стала равной 1000. Индукция магнитного поля в данном металлическом бруске равна

A) 0,6 Тл. B) 2,4 Тл. C) 0,8 Тл. D) 1,4 Тл. E) 0,4Тл.

7. Магнитный поток.

1. Если вектор индукции магнитного поля В образует угол α cплоскостью рамки, то магнитный поток через площадь S плоского витка равен.

A) BS. B) BScosα. C) BS/sinα. D) BS/cosα. E) BSsinα.

2. Магнитный поток, создаваемый током I в контуре равен.

A) ΔФ = — ЕiΔt. B) Ф = BS. C) Ф = BS sin(wt). D) Ф = LI. E) Ф = BScosα.

3. Единица магнитного потока.

A) Тл. B) Вб. C) А·м2. D) А/м. E) Гн.

4. Магнитный поток через контур индуктивностью 4 Гн при силе тока 2 А равен.

A) 0,5 Вб. B) 8 Вб. C) 2 Вб. D) 4 Вб. E) 1 Вб.

Решите задачи:

5. Магнитный поток проходит сквозь солнечное пятно площадью 1,2 ∙ 1015 м2. Если средняя индукция магнитного поля пятна равна 0,3 Тл, а линии индукции магнитного поля пятна перпендикулярны его поверхности, то магнитный поток равен

A) 4·1014 Вб. B) 0,4·1014 Вб. C) 3,6·1014 Вб. D) 36·1014 Вб. E) 0,25·10-15 Вб.

6. Контур площадью 100 см ² находится в однородном магнитном поле с индукцией 2 Тл. Если плоскость контура перпендикулярна вектору индукции, то пронизывающий его магнитный поток равен.

A) 200 Вб. B) 0 Вб. C) 20 Вб. D) 2·10² Вб. E) 2 Вб.

7. Прямоугольная рамка со сторонами 25 см и 60 см пересекается магнитным полем с магнитной индукцией 1,5 Тл. Если вектор В образует с нормалью к рамке угол 0º, то магнитный поток, пронизывающий рамку, равен.

A) ≈ 23 Вб. B) ≈ 0,13 Вб. C) ≈ 0,15 Вб. D) ≈ 0,10 Вб. E) ≈ 0,23 Вб.

8. В постоянное однородное магнитное поле перпендикулярно силовым линиям вектора магнитной индукции В поместили квадрат со стороной а. При этом магнитный поток Ф через площадь квадрата равен.

A) 4а2·В. B) В/4а2. C) В·а2. D) В·а. E) В/а2.

9. Плоский контур, расположенный перпендикулярно вектору индукции магнитного поля, пронизывает магнитный поток 2 Вб. Если площадь контура 4 м2, то индукция магнитного поля равна.

A) 2 Тл. B) 1 Тл. C) 6 Тл. D) 0,5 Тл. E) 8 Тл.

8. Поток магнитной индукции. Закон ЭМИ, правило Ленца.

1. При выдвигании из катушки постоянного магнита в ней возникает электрический ток. Это — явление

A) электростатической индукции. B) электромагнитной индукции.

C) самоиндукции. D) индуктивности. E) магнитной индукции.

2. Контур находится в переменном магнитном поле, изменяющемся со скоростью

4 Вб/с. ЭДС индукции в проводящем контуре равна

A) 0,1 В. B) 0,4 В. C) 2 В. D) 0,2 В. E) 4 В.

3. Закон электромагнитной индукций

A) B) C) D) E)

4. За 4 с магнитный поток, пронизывающий контур, равномерно уменьшился с 19 Вб до 7 Вб. ЭДС индукции в контуре равна

A) 3 В. B) 4 В. C) 8 В. D) 11 В. E) 12 В.

5. Скорость изменения магнитного потока через контур равна 2 Вб/с. ЭДС индукции в данном контуре

A) 4 В. B) 2 В. C) 1 В. D) 0,5 В. E) 8 В.

6. Скоростью изменения магнитного потока через контур определяется.

A) индуктивность контура. B) магнитная индукция. C) ЭДС самоиндукции.

D) электрическое сопротивление контура. E) ЭДС индукции в замкнутом контуре.

7. Направление индукционного тока определяется по

A) правилу Ленца. B) закону Кулона. C) закону сохранения энергии. D) правилу левой руки. E) закону сохранения электрического заряда.

Решите задачи:

8. Скорость изменения магнитного потока через контур равна 2 Вб/с ЭДС индукции в данном контуре.

A) 4 В. B) 2 В. C) 1 В. D) 0,5В. E) 8 В.

9. Меняющийся магнитный поток, пронизывающий контур сопротивлением 0,2 Ом, создает в контуре ток 4 А. Если изменение магнитного потока равна 0,4 Вб, то время

A) 0,5 с. B) 0,8 с. C) 2 с. D) 1 с. E) 1,6 с.

10. Скорость изменения магнитного потока равна 4 Вб/с. ЭДС индукции в катушке, если она имеет 100 витков, равна.

A) 400 В. B) 0,04 В. C) 25 В. D) 2 В. E) 4 В.

11. В замкнутом витке проводника сопротивлением 2∙10-2 Ом мгновенное значение индукционного тока равно 0,5 А. ЭДС индукции равна

A) 0,01 В. B) 2 В. C) 0,1 В. D) 10 В. E) 1 В.

12. При равномерном исчезновении магнитного поля в течение 0,5 с, в катушке, содержащей 800 витков, индуцируется ЭДС 40 В. Первоначальное значение магнитного потока равно.

A) 4 Вб. B) 2 мВб. C) 0,32Вб. D) 25 мВб. E) 0,5 Вб.

13. В замкнутом витке проводника сопротивлением 2·10‾²Ом мгновенное значение индукционного тока равно 5А. ЭДС индукции, равна.

A) 10 В. B) 0,1 В. C) 1 В. D) 2 В. E) 0,2 В.

14. Заряд, прошедший через поперечное сечение витка, сопротивление которого

0,03 Ом, при уменьшении магнитного потока внутри витка на 12 мВб за 1 секунду, равен

A) 0,2 Кл. B) 400 Кл. C) 2 Кл. D) 0,4 Кл. E) 4 Кл.

15. За 0,2 с магнитный поток, пронизывающий контур, равномерно уменьшился с

3 до 1 Вб. При этом ЭДС индукции в контуре равна.

A) 0,8 В. B) 20 В. C) 25 В. D) 10 В. E) 15 В.

16. В замкнутом витке проводника сопротивлением 2∙10-2 Ом мгновенное значение индукционного тока равно 0,5А. ЭДС индукции равна

A) 0,01 В. B) 2 В. C) 0,1 В. D) 10 В. E) 1 В.

*17. В однородном магнитном поле с индукцией 5·10-3Тл, находится прямоугольная рамка. Сопротивление рамки равно 0,5 Ом. Вначале плоскость рамки составляла угол 30º с вектором индукции магнитного поля. Затем рамку повернули так, что вектор индукции стал параллелен плоскости рамки, при этом через рамку прошел заряд 5 мкКл. Площадь рамки равна (sin 30º = 0,5)

A) 8 см2. B) 20 см2. C) 10 см2. D) 12 см2. E) 15 см2.

*18.Магнитный поток в рамке, равномерно вращающейся в однородном магнитном поле, изменяется по закону Ф=3·10-2cos157t. Уравнение зависимости мгновенного значения ЭДС индукции от времени имеет вид

A) 15,7sin157t . B) 0,03sin157t. C) 0,3sin157t. D) 47,1sin157t. E) 4,71sin157t.

*19. Замкнутый проводник сопротивлением 43 Ом находится в магнитном поле. В результате изменения магнитного поля магнитный потом через проводник возрос с 0,0002 Вб до 0,0005 Вб. Через поперечное сечение проводника при этом прошел заряд

A) 10-5 Кл. B) 104 Кл. C) 10-4 Кл. D) 105 Кл. E) 10-6 Кл.

*20. При изменении магнитного потока, пронизывающего замкнутый контур в зависимости от времени, как показано на графике, максимальный модуль ЭДС индукции, возникающей в контуре, наблюдается в промежуток времени

A) 0 — 4 с. B) 4 с — 6 с. C) 4 с – 8 с. D) 8 с- 9 с. E) 9 с – 12 с.

*21. Зависимость от времени магнитного потока, пронизывающего виток, показана на рисунке. Если сопротивление витка равно 0,2 Ом, то в витке на интервале ВС равен

A) 12 мА. B) 4 А. C) 8 мА. D) 2 мА. E) 6 мА.

9. ЭДС в движущихся проводниках.

Решите задачи:

1. Самолет летит горизонтально, его скорость 900 км/ч. Если вертикальная составляющая земного магнитного поля равна 50 мкТл, а размах крыльев самолета 12 м, то разность потенциалов, возникающая между концами крыльев самолета, равна.

A) 0,15В. B) 1,5В. C) 150 В. D) 1500 В. E) 15 В.

2. Вектор магнитной индукции перпендикулярен проводнику и по модулю равен

2 Тл. ЭДС индукции в проводнике длиной 1 м, движущимся со скоростью 20 м/с равна

A) 1 В. B) 0. C) 10 В. D) 0,1 В. E) 40 В.

3. Проводник длиной 2 м движется со скоростью 10 м/с в магнитном поле с индукцией 2 Тл перпендикулярно линиям магнитной индукции. В нем индуцируется ЭДС, равная.

A) 0,4 В. B) 0. C) 2,5 В. D) 10 В. E) 40 В.

4. Магнитный поток 4 Вб можно создать контуром индуктивностью 2 Гн при силе тока в нем

A) 0,5 А. B) 10 А. C) 4 А. D) 8 А. E) 2 А.

*5. В однородном магнитном поле перпендикулярно линиям магнитной индукции расположен круговой виток площадью 400 см2. Если за 2 секунды индукция магнитного поля равномерно изменилась от 0,6 Тл до 0,1 Тл, то в контуре при этом возникла ЭДС

A) 40 мВ. B) 10 мВ. C) 50 мВ. D) 80 мВ. E) 20 мВ.

*6. Поезд движется по рельсам, расстояние между которыми 1,2 м, со скоростью

72 км/ч. Вертикальная составляющая индукция магнитного поля Земли 50 мкТл. Если на рельсы положить перемычку сопротивлением 0,2 Ом, то по ней потечет ток величиной

A) 6 мА. B) 1 А. C) 1,44 мА. D) 0,24 А. E) 3,6 А.

10. ЭДС самоиндукции, индуктивность.

1. Единицей индуктивности является 1 Гн, который равен.

A) 1 Тл·А·м2. B) 1 Тл/А2. C) 1 Вб/А. D) 1 Вб·А. E) 1 А·м2.

2. Индуктивность измеряется в

A) Тл. B) Вб. C) Гн. D) Дж. E) Ф.

Решите задачи:

3. Сила тока в катушке равномерно изменилась от 0 до 10 А за 0,1 с. Если при этом в катушке в ней возникла ЭДС самоиндукции 60 В, то индуктивность катушки.

A) 0. B) 0,8 Гн. C) 0,6 Гн. D) 0,9 Гн. E) 6 Гн.

4. Контур находиться в переменном магнитном поле, изменяющемся со скоростью

4 Вб/с. ЭДС индукции в проводящем контуре равна.

A) 0,1 В. B) 0,4 В. C) 4 В. D) 0,2 В. E) 2 В.

5. В катушке с индуктивностью 68 мГн сила тока 3,8 А исчезает за 0,012 с. ЭДС самоиндукции равна.

A) ≈0. B) ≈21,5. C) ≈0,67. D) ≈0,21. E) ≈3,1.

6. Если при силе тока 4 А в контуре существует магнитный поток 2 Вб, то индуктивность контура равна.

A) 10 Гн. B) 1 Гн. C) 2 Гн. D) 18 Гн. E) 0,5 Гн.

7. ЭДС самоиндукции соленоида индуктивность 0,4 Гн при равномерном изменении силы тока в ней на 10 А за 0,2 с равно.

A) 10 В. B) 50 В. C) 30 В. D) 40 В. E) 20 В.

*8. Плоская рамка площадью 4·10-4 м2 расположена в магнитном поле так, что нормаль к рамке составляет с направлением вектора магнитной индукции поля угол 600. Индукция магнитного поля, пронизывающего рамку, изменяется по закону

В = 0,05t. По истечении 4 с ЭДС индукции, возникающая в рамке, равна

(cos 600 = 0,5)

А) 10-3 В. В) 10-5 В. С) 10-2 В. D) 10-6 В. Е) 10-4 В.

*9. Замкнутая накоротко катушка диаметром 10 см, имеющая 20 витков, находится в магнитном поле, индукция которого увеличивается от 2 до 6 Тл в течение 0,1 с. Если плоскость витков перпендикулярна силовым линиям поля, то среднее значение ЭДС индукции катушки равно (π ≈ 3,14)

A) 6,28 В. B) 0,82 В. C) 6,2 В. D) 8,2 В. E) 0,62 В.

11. Энергия и плотность энергии МП.

1. Энергия магнитного поля, создаваемого током в замкнутом контуре индуктивностью L, равна

A) W=CU/2. B) W=q2/2C. C) W=mgh. D) W=mυ2/2. E) W=LI2/2.

2. Через катушку индуктивностью 3 Гн протекает постоянный ток. Если сила тока в этой цепи равна 4 А, то энергия магнитного поля катушки равна

A) 48 Дж. B) 36 Дж. C) 6 Дж. D) 24 Дж. E) 12 Дж.

3. Сила тока в катушке увеличилась с 1 до 3 А. Энергия магнитного поля катушки.
A) не изменилась. B) увеличилась в 4 раза. C) увеличилась в 9 раз.

D) увеличилась в 3 раза. E) увеличилась в 16 раз.

4. Чтобы при неизменном значении силы тока в контуре энергия магнитного поля уменьшилась в 4 раза, индуктивность нужно.

A)уменьшить в 4 раза. B) уменьшить в 8 раз. C) уменьшить в 16 раз.

D) уменьшить в 2 раза. E) уменьшить в 24 раза.

5. Если энергия магнитного поля контура уменьшилась в 4 раза, то сила тока в нём.

A) уменьшилась в 16 раз. B) увеличилась в 4 раза. C) увеличилась в 16 раз.

D) увеличилась в 2 раза. E) уменьшилась в 2 раза.

6. Энергия магнитного поля катушки индуктивностью 2 Гн при силе тока 200 мА равна.

A) 400 Дж. B) 4·104 Дж. C) 0,4 Дж. D) 4·102 Дж. E) 8·10-2 Дж.

7. Энергия магнитного поля катушки индуктивностью 5 Гн при силе тока в ней

400 мА равна

A) 2 Дж. B) 1 Дж. C) 0,8 Дж. D) 0,4 Дж. E) 4·10 Дж.

1. МП и магнитное взаимодействие.

1.Д

2.Д

3.С

4.С

5.Д

6.Д

7.А

8.В

9. Д

2. Вектор магнитной индукции магнитного поля.

1.А

2.С

3.Е

4.Д

3. Силы Ампера.

1.Д

2.С

3.В

4.А

5.Е

6.Д

7.В

4. Силы Лоренца.

1.А

2.Д

3.А

4.В

5.Е

6.Д

7.Е

8.Д

9.А

10.С

11.Е

12.С

5. Движение заряженной частицы в магнитном поле.

1.А

2.В

3.Е

4.В

5.А

6.Д

7.Д

8.В

9.А

10.В

11.С

12.С

6. Магнитные свойства.

1.Д

2.В

3.Е

4.А

5.Е

6.А

7.В

8.Д

9.Е

1.Д

7. Магнитный поток.

1.Е

2.Д

3.В

4.В

5.С

6.Д

7.Е

8.С

9.Д

8. Поток магнитной индукции. Закон ЭМИ, правило Ленца.

1.В

2.Е

3.Е

4.А

5.В

6.Е

7.А

8.В

9.А

10.А

11.А

12.Д

13.В

14.Д

15.Д

16.А

17.С

18.Е

19.С

20.Д

21.В

9. ЭДС в движущихся проводниках.

1.А

2.Е

3.Е

4.Е

5.В

6.А

10. ЭДС самоиндукции, индуктивность.

1.С

2.С

3.С

4.С

5.В

6.Е

7.Е

8.В

9.А

11. Энергия и плотность энергии МП.

1.Е

2.Д

3.С

4.А

5.Е

6.Д

7.Д

Электромагнитная индукция. Правило Ленца

Явление электромагнитной индукции заключается в том, что в результате изменения во времени магнитного потока, который пронизывает замкнутый проводящий контур, в контуре возникает электрический ток. Открыто это явление было физиком из Великобритании Максом Фарадеем в 1831 году.

Формула магнитного потока

Введем обозначения, необходимые нам для записи формулы. Для обозначения магнитного потока используем букву Ф, площади контура – S, модуля вектора магнитной индукции – B, α – это угол между вектором B→ и нормалью n→ к плоскости контура.

Магнитный поток, который проходит через площадь замкнутого проводящего контура, можно задать следующей формулой:

Φ=B·S·cos α,

Проиллюстрируем формулу.

Рисунок 1.20.1. Магнитный поток через замкнутый контур. Направление нормали n→ и выбранное положительное направление l→ обхода контура связаны правилом правого буравчика.

За единицу магнитного потока в СИ принят 1 вебер (Вб). Магнитный поток, равный 1 Вб, может быть создан в плоском контуре площадью 1 м2 под воздействием магнитного поля с индукцией 1 Тл, которое пронизывает контур по направлению нормали.

1 Вб=1 Тл·м2

Закон Фарадея

Изменение магнитного потока приводит к тому, что в проводящем контуре возникает ЭДС индукции δинд. Она равна скорости, с которой происходит изменение магнитного потока через ограниченную контуром поверхность, взятой со знаком минус. Впервые экспериментально установил это Макс Фарадей. Он же записал свое наблюдение в виде формулы ЭДС индукции, которая теперь носит название Закона Фарадея:

Определение 1

Закон Фарадея:

δинд=-∆Φ∆t

Правило Ленца

Определение 2

Согласно результатам опытов, индукционный ток, который возникает в замкнутом контуре в результате изменения магнитного потока, всегда направлен определенным образом. Создаваемое индукционным током магнитное поле препятствует изменению вызвавшего этот индукционный ток магнитного потока. Ленц сформулировал это правило в 1833 году.

Проиллюстрируем правило Ленца рисунком, на котором изображен неподвижный замкнутый проводящий контур, помещенный в однородное магнитное поле. Модуль индукции увеличивается во времени. 

Пример 1

Рисунок 1.20.2. Правило Ленца

Здесь ∆Φ∆t>0, а δинд<0 < 0. Индукционный ток Iинд протекает навстречу выбранному положительному направлению l→ обхода контура.

Благодаря правилу Ленца мы можем обосновать тот факт, что в формуле электромагнитной индукции δинд и ∆Φ∆t противоположны по знакам.

Если задуматься о физическом смысле правила Ленца, то это частный случай Закона сохранения энергии.

Нужна помощь преподавателя?

Опиши задание — и наши эксперты тебе помогут!

Описать задание

Причины возникновения индукционного тока в движущихся и неподвижных проводниках

Причин, по которым может происходить изменение магнитного потока, пронизывающего замкнутый контур, две:

  1. Изменение магнитного потока вследствие перемещения всего контура или отдельных его частей в магнитном поле, которое не изменяется со временем;
  2. Изменение магнитного поля при неподвижном контуре.

Перейдем к рассмотрению этих случаев подробнее.

Перемещение контура или его частей в неизменном магнитном поле

При движении проводников и свободных носителей заряда в магнитном поле возникает ЭДС индукции. Объяснить возникновение δинд можно действием силы Лоренца на свободные заряды в движущихся проводниках. Сила Лоренца здесь – это сторонняя сила.

Пример 2

На рисунке мы изобразили пример индукции, когда прямоугольный контур помещен в однородное магнитное поле B→ направленное перпендикулярно плоскости контура. Одна из сторон контура перемещается по двум другим сторонам с некоторой скоростью.

Рисунок 1.20.3. Возникновение ЭДС индукции в движущемся проводнике. Отражена составляющая силы Лоренца, которая действует на свободный электрон

На свободные заряды подвижной части контура воздействует сила Лоренца. Основная составляющая силы Лоренца в данном случае направлена вдоль проводника и связана с переносной скоростью зарядов υ→. Модуль этой сторонней силы равен:

FЛ=eυ→B.

Работа силы FЛ на пути l равна:

A=FЛ·l=eυBl.

По определению ЭДС: 

δинд=Ae=υBl.

Значение сторонней силы для неподвижных частей контура равно нулю. Для соотношения δинд можно записать другой вариант формулы. Площадь контура с течением времени изменяется на ΔS=lυΔt. Соответственно, магнитный поток тоже будет с течением времени изменяться: ΔΦ=BlυΔt.

Следовательно, 

δинд=∆Φ∆t.

Знаки в формуле, которая связывает δинд и ∆Φ∆t, можно установить в зависимости от того, какие направления нормали и направления контура будут выбраны. В случае выбора согласованных между собой по правилу правого буравчика направлений нормали n→ и положительного направления обхода контура l→ можно прийти к формуле Фарадея.

При условии, что сопротивление всей цепи – это R, то по ней будет протекать индукционный ток, который равен Iинд=δиндR. За время Δt на сопротивлении R выделится джоулево тепло:

∆Q=RIинд2∆t=υ2B2l2R∆t

Парадокса здесь нет. Мы просто не учли воздействие на систему еще одной силы. Объяснение заключается в том, что при протекании индукционного тока по проводнику, расположенному в магнитном поле, на свободные заряды действует еще одна составляющая силы Лоренца, которая связана с относительной скоростью движения зарядов вдоль проводника. Благодаря этой составляющей появляется сила Ампера FА→.

Для рассмотренного выше примера модуль силы Ампера равен FA =IBl. Направление силы Ампера таково, что она совершает отрицательную механическую работу Aмех. Вычислить эту механическую работу за определенный период времени можно по формуле:

Aмех=-Fυ∆t=-IBlυ∆t=-υ2B2l2R∆t

Проводник, перемещающийся в магнитном поле, испытывает магнитное торможение. Это приводит к тому, что полная работа силы Лоренца равна нулю. Джоулево тепло может выделяться либо за счет уменьшения кинетической энергии движущегося проводника, либо за счет энергии, которая поддерживает скорость перемещения проводника в пространстве.

Изменение магнитного поля при неподвижном контуре

Определение 3

Вихревое электрическое поле – это электрическое поле, которое вызывается изменяющимся магнитным полем.

В отличие от потенциального электрического поля работа вихревого электрического поля при перемещении единичного положительного заряда по замкнутому проводящему контуру равна δинд в неподвижном проводнике.

В неподвижном проводнике электроны могут приводиться в движение только под действием электрического поля. А возникновение δинд нельзя объяснить действием силы Лоренца.

Первым, кто ввел понятие вихревого электрического поля, был английский физик Джон Максвелл. Случилось это в 1861 году.

Фактически, явления индукции в подвижных и неподвижных проводниках протекают одинаково. Так что в этом случае мы тоже можем использовать формулу Фарадея. Отличия касаются физической причины возникновения индукционного тока: в движущихся проводниках δинд обусловлена силой Лоренца, в неподвижных – действием на свободные заряды вихревого электрического поля, возникающего при изменении магнитного поля.

Рисунок 1.20.4. Модель электромагнитной индукции

Рисунок 1.20.5. Модель опытов Фарадея

Рисунок 1.20.6. Модель генератора переменного тока

Физика 11 класс. Законы, правила, формулы

Физика 11 класс. Законы, правила, формулы | Задачи по физике

Перейти к содержимому

    Магнитное поле

  • Модуль вектора магнитной индукции
    Модуль вектора магнитной индукции (В) — это отношение максимальной силы (Fmax), действующей со стороны маг нитного поля на участок проводника с током, к произведению силы тока (I) на длину (l) этого участка.

    СИ: Тл

  • Закон Ампера
    Сила Ампера (FА) — это сила, действующая на участок проводника с током в магнитном поле, равная произведению вектора магнитной индукции (В) на силу тока (I), длину участка (l) проводника и на синус угла (α) между магнитной индукции и участком проводника.

    СИ: Н

  • Сила Лоренца
    Сила Лоренца — это сила (FЛ), действующая на движущуюся заряженную частицу со стороны магнитного поля, равная произведению модуля вектора магнитной индукции (В) на заряд частицы (q), на скорость (v) её упорядоченного движения в проводнике и на синус угла (α) между вектором скорости и вектором магнитной индукции.
    FЛ=q×v×B×sinα
    СИ: Н
  • Движение заряженной частицы в магнитном поле
    В однородном магнитном поле (В), направленном перпендикулярно к начальной скорости (v) частицы массой (m) с зарядом (q), сама частица равномерно движется по окружности радиусом (r) с период обращения (T).
    ,

    СИ: м, с

  • Магнитная проницаемость среды
    Магнитная проницаемость (μ) — это величина, характеризующая магнитные свойства среды и равная отношению вектора магнитной индукции (В) в однородной среде к вектору магнитной индукции (В0) в вакууме.
    Механические колебания и волны

  • Период колебаний
    Период колебаний (Т) — продолжительность одного полного колебания, определяемая как отношение времени (t), за которое совершено (N) полных колебаний, к числу этих колебаний

    СИ: с

  • Частота колебаний
    Частота колебаний (ν) — число колебаний в единицу времени, равное величине, обратной периоду колебаний (Т).

    СИ: с-1

  • Циклическая (круговая частота)
    Циклическая (круговая) частота (ω) показывает, какое число колебаний совершает тело за единиц времени, и связана с периодом (T) и частотой (ν) колебаний зависимостями:
    ;

    СИ: рад/с

  • Период колебаний пружинного маятника
    Период колебаний (T) пружинного маятника тем больше, чем больше масса тела (m) и тем меньше, чем больше жесткость пружины (k).

    СИ: с

  • Собственная частота колебательной системы
    1) пружинного маятника (ωп): ωп=
    2) математического маятника (ωм): ωм=
    СИ: рад/с
  • Гармонические колебания
    1) Уравнение гармонических колебаний (уравнение координат колеблющегося тела): ; ;
    2) Уравнение скорости колеблющегося тела: ; ;
    3) Уравнение ускорения колеблющегося тела: ; .
    СИ: м, м/с, м/с2
  • Полная механическая энергия колеблющегося пружинного маятника
    Полная механическая энергия (W) колеблющегося тела равна:
    1) сумме кинетической (WК) и потенциальной (WП) энергий в каждый момент времени: W=WК+WП= ;
    2) половине произведения квадрата амплитуды (А) (максимальной координаты x=xmax) его колебаний и жесткости пружины (k): W= WПmax= ;
    3) половине произведения квадрата максимальной скорости (vmax) и массы (m) тела: WКmax= .
    СИ: Дж
  • Скорость волны
    Скорость волны (v) (скорость распространения колебаний в пространстве) равна произведению частоты колебаний (ν) в волне на длину волны (λ).

    СИ: м/с

  • Длина волны
    Длина волны (λ) — расстояние, на которое распространяются колебания со скоростью (v) за время, равное периоду колебаний (T).

    СИ: м

    Электромагнитные колебания

  • Полная энергия колебательного контура
    Полная энергия (W) электромагнитного поля контура равна сумме энергий магнитного и эле

7 «Б»














Урок




1/1

 Что изучает физика. Физические термины. Наблюдения и опыты.§ 1 — 3, Л № 5, 12
2/2 Физические величины. Измерение физических величин. Погрешность и точность измерений§ 4, 5, упр.1
3/3 Определение цены деления измерительного прибора§ 4, 5
4/4 Физика и техника§ 6,
  Первоначальные сведения о строении вещества 
5/1 Строение вещества. Молекулы§ 7, 8
6/2 Определение размеров малых тел§ 7, 8
7/3 Движение молекул. Диффузия в газах, жидкостях и твердых телах§ 9,
8/4 Взаимодействие молекул

9/5

 Три состояния вещества§ 11, 12
10/6 Повторение. Контрольная работа №1 «Первоначальные сведения о строении веществ໧ 12
   

Электромагнитная индукция и закон Фарадея

Если затем намотать провод в катушку, магнитное поле значительно усиливается, создавая вокруг себя статическое магнитное поле, формирующее форму стержневого магнита, дающего отчетливые северный и южный полюсы.

Полая катушка с воздушным сердечником

Магнитный поток, развиваемый вокруг катушки, пропорционален величине тока, протекающего в обмотках катушек, как показано. Если дополнительные слои проволоки намотаны на ту же катушку с тем же током, протекающим через них, напряженность статического магнитного поля будет увеличена.

Следовательно, напряженность магнитного поля катушки определяется ампер витков катушки. Чем больше витков провода внутри катушки, тем больше напряженность статического магнитного поля вокруг нее.

Но что, если бы мы изменили эту идею, отключив электрический ток от катушки и вместо полого сердечника мы поместили стержневой магнит внутри сердечника катушки с проволокой. Перемещая этот стержневой магнит «внутрь» и «наружу» из катушки, в катушку будет индуцироваться ток за счет физического движения магнитного потока внутри нее.

Аналогичным образом, если бы мы удерживали стержневой магнит в неподвижном состоянии и перемещали катушку вперед и назад в магнитном поле, в катушке индуцировался бы электрический ток. Затем, перемещая провод или изменяя магнитное поле, мы можем индуцировать напряжение и ток внутри катушки, и этот процесс известен как Электромагнитная индукция и является основным принципом работы трансформаторов, двигателей и генераторов.

Электромагнитная индукция была впервые обнаружена еще в 1830-х годах Майклом Фарадеем .Фарадей заметил, что когда он перемещал постоянный магнит внутрь и из катушки или одиночной петли провода, он индуцировал электродвижущую силу или ЭДС, другими словами, напряжение, и, следовательно, создавался ток.

Итак, Майкл Фарадей открыл способ создания электрического тока в цепи с использованием только силы магнитного поля, а не батарей. Затем это привело к очень важному закону, связывающему электричество с магнетизмом, Закон электромагнитной индукции Фарадея .Так, как это работает?.

Когда магнит, показанный ниже, перемещается «к» катушке, стрелка или стрелка гальванометра, который в основном представляет собой очень чувствительный амперметр с подвижной катушкой с нулевым центром, отклоняется от своего центрального положения только в одном направлении. Когда магнит перестает двигаться и остается неподвижным по отношению к катушке, стрелка гальванометра возвращается к нулю, поскольку нет физического движения магнитного поля.

Аналогично, когда магнит перемещается «от катушки» в другом направлении, стрелка гальванометра отклоняется в противоположном направлении относительно первого, указывая на изменение полярности.Затем, перемещая магнит назад и вперед по направлению к катушке, стрелка гальванометра будет отклоняться влево или вправо, положительно или отрицательно, относительно направленного движения магнита.

Электромагнитная индукция движущимся магнитом

Аналогичным образом, если магнит теперь удерживается в неподвижном состоянии и ТОЛЬКО катушка перемещается к магниту или от него, стрелка гальванометра также будет отклоняться в любом направлении. Затем движение катушки или петли через магнитное поле индуцирует в катушке напряжение, величина которого пропорциональна скорости или скорости движения.

Тогда мы видим, что чем быстрее движется магнитное поле, тем больше будет индуцированная ЭДС или напряжение в катушке, поэтому для выполнения закона Фарадея должно быть «относительное движение» или движение между катушкой и магнитным полем. и либо магнитное поле, либо катушка, либо и то, и другое могут двигаться.

Закон индукции Фарадея

Из приведенного выше описания мы можем сказать, что существует взаимосвязь между электрическим напряжением и изменяющимся магнитным полем, о которой говорится в известном законе Майкла Фарадея об электромагнитной индукции: «что напряжение индуцируется в цепи всякий раз, когда существует относительное движение между проводником и проводником. магнитное поле и что величина этого напряжения пропорциональна скорости изменения магнитного потока ».

Другими словами, Электромагнитная индукция — это процесс использования магнитных полей для создания напряжения, а в замкнутой цепи — тока.

Итак, какое напряжение (ЭДС) может быть наведено в катушку, используя только магнетизм. Это определяется следующими тремя различными факторами.

  • 1). Увеличение количества витков провода в катушке — за счет увеличения количества отдельных проводников, прорезающих магнитное поле, количество создаваемой наведенной ЭДС будет суммой всех отдельных витков катушки, поэтому, если есть 20 витков в в катушке будет в 20 раз больше наведенной ЭДС, чем в одном куске проволоки.
  • 2). Увеличение скорости относительного движения между катушкой и магнитом — если одна и та же катушка с проволокой прошла через то же магнитное поле, но ее скорость или скорость увеличились, проволока перережет линии потока с большей скоростью, что приведет к большей наведенной ЭДС будет произведено.
  • 3). Увеличение силы магнитного поля. Если ту же катушку с проволокой перемещать с той же скоростью через более сильное магнитное поле, будет возникать большая ЭДС, потому что есть больше силовых линий, которые нужно разрезать.

Если бы мы могли перемещать магнит на диаграмме выше в катушку и из нее с постоянной скоростью и на постоянное расстояние без остановки, мы бы генерировали непрерывно индуцированное напряжение, которое чередовалось бы между одной положительной полярностью и отрицательной полярностью, создавая переменную или отрицательную полярность. Выходное напряжение переменного тока, и это основной принцип работы электрического генератора, аналогичного тем, которые используются в динамо-машинах и автомобильных генераторах.

В небольших генераторах, таких как велосипедные динамо-машины, небольшой постоянный магнит вращается под действием велосипедного колеса внутри фиксированной катушки.В качестве альтернативы, электромагнит, питаемый фиксированным постоянным напряжением, может вращаться внутри фиксированной катушки, например, в больших генераторах энергии, вырабатывающих в обоих случаях переменный ток.

Простой генератор с использованием магнитной индукции

Простой генератор динамо-типа, представленный выше, состоит из постоянного магнита, который вращается вокруг центрального вала, и катушки с проволокой, расположенной рядом с этим вращающимся магнитным полем. Когда магнит вращается, магнитное поле вокруг верхней и нижней части катушки постоянно меняется между северным и южным полюсами.Это вращательное движение магнитного поля приводит к тому, что в катушке индуцируется переменная ЭДС, как определено законом электромагнитной индукции Фарадея.

Величина электромагнитной индукции прямо пропорциональна плотности потока β, количеству петель, определяющих общую длину проводника, l в метрах, и скорости или скорости, ν, с которой изменяется магнитное поле внутри проводника в метрах / секунда или м / с, что выражается выражением двигательной ЭДС:

Движение ЭДС Фарадея, выражение

Если проводник не движется под прямым углом (90 °) к магнитному полю, то к приведенному выше выражению будет добавлен угол θ °, что приведет к уменьшению выходной мощности при увеличении угла:

Закон электромагнитной индукции Ленца

Закон Фарадея говорит нам, что наведение напряжения в проводник может быть выполнено либо путем пропускания его через магнитное поле, либо путем перемещения магнитного поля мимо проводника, и что если этот проводник является частью замкнутой цепи, электрический ток будет течь. .Это напряжение называется индуцированной ЭДС , поскольку оно было индуцировано в проводнике изменяющимся магнитным полем из-за электромагнитной индукции с отрицательным знаком в законе Фарадея, указывающим нам направление индуцированного тока (или полярность наведенной ЭДС).

Но изменяющийся магнитный поток создает переменный ток через катушку, которая сама создает собственное магнитное поле, как мы видели в учебнике по электромагнитам. Эта самоиндуцированная ЭДС противодействует вызывающему ее изменению, и чем выше скорость изменения тока, тем больше противоположная ЭДС.Эта самоиндуцированная ЭДС по закону Ленца будет противодействовать изменению тока в катушке, и из-за ее направления эту самоиндуцированную ЭДС обычно называют обратной ЭДС .

Закон Ленца гласит: «Направление индуцированной ЭДС таково, что она всегда будет противодействовать изменению, которое ее вызывает». Другими словами, индуцированный ток всегда будет ПРОТИВ движению или изменению, которое в первую очередь привело к возникновению индуцированного тока, и эта идея обнаруживается в анализе индуктивности.

Аналогично, если магнитный поток уменьшается, то наведенная ЭДС будет противодействовать этому уменьшению, создавая индуцированный магнитный поток, который добавляется к исходному потоку.

Закон Ленца является одним из основных законов электромагнитной индукции для определения направления потока индуцированных токов и связан с законом сохранения энергии.

Согласно закону сохранения энергии, который гласит, что общее количество энергии во Вселенной всегда будет оставаться постоянным, поскольку энергия не может быть создана или уничтожена.Закон Ленца выводится из закона индукции Майкла Фарадея.

Последний комментарий к закону Ленца об электромагнитной индукции. Теперь мы знаем, что когда существует относительное движение между проводником и магнитным полем, внутри проводника индуцируется ЭДС.

Но проводник на самом деле может не быть частью электрической цепи катушек, но может быть железным сердечником катушек или какой-либо другой металлической частью системы, например, трансформатором. Индуцированная ЭДС внутри этой металлической части системы заставляет циркулирующий ток течь вокруг нее, и этот тип тока сердечника известен как вихревой ток .

Вихревые токи, генерируемые электромагнитной индукцией, циркулируют вокруг сердечника катушек или любых соединяющих их металлических компонентов внутри магнитного поля, поскольку для магнитного потока они действуют как одиночная петля из проволоки. Вихревые токи ничего не вносят в полезность системы, но вместо этого они противодействуют потоку индуцированного тока, действуя как отрицательная сила, вызывая резистивный нагрев и потери мощности внутри сердечника. Однако существуют электромагнитные индукционные печи, в которых для нагрева и плавления ферромагнитных металлов используются только вихревые токи.

Вихревые токи, циркулирующие в трансформаторе

Изменяющийся магнитный поток в железном сердечнике трансформатора выше будет индуцировать ЭДС не только в первичной и вторичной обмотках, но и в железном сердечнике. Железный сердечник является хорошим проводником, поэтому токи, индуцируемые в твердом железном сердечнике, будут большими. Кроме того, вихревые токи текут в направлении, которое, согласно закону Ленца, ослабляет поток, создаваемый первичной катушкой. Следовательно, ток в первичной катушке, необходимый для создания заданного поля B, увеличивается, поэтому кривые гистерезиса становятся более толстыми по оси H.

Ламинирование железного сердечника

Потери на вихревые токи и гистерезис нельзя полностью исключить, но их можно значительно уменьшить. Вместо использования твердого железного сердечника в качестве материала магнитного сердечника трансформатора или катушки магнитный путь является «ламинированным».

Эти пластинки представляют собой очень тонкие полосы изолированного (обычно покрытого лаком) металла, соединенные вместе для образования твердого сердечника. Пластины увеличивают сопротивление железного сердечника, тем самым увеличивая общее сопротивление потоку вихревых токов, поэтому индуцированные потери мощности на вихревые токи в сердечнике уменьшаются, и именно по этой причине цепь магнитного железа трансформаторов и электрические машины все ламинированы.

электромагнетизм — Какова скорость магнитного поля?

Я утверждаю, что единственный способ создать магнитное поле — это движущиеся электрические заряды. Это утверждение может быть ошибочным — кто-нибудь может когда-нибудь найти другой способ. Утверждают, что магнитные поля от магнитов могут возникать из-за вращения неподвижных электрических зарядов. У меня нет мнения об этом.Я собираюсь использовать движущиеся электрические заряды, потому что это то, что я знаю. Я не знаю ни о каком источнике, кроме движущихся или вращающихся зарядов.

Магнитное поле можно обнаружить по его влиянию на другие движущиеся электрические заряды. Если исходный заряд или целевой заряд не движутся — если один из них неподвижен — тогда цель не обнаруживает магнитный заряд. (На данный момент я игнорирую их ускорение.)

Поскольку они оба должны двигаться, магнитное поле можно обнаружить только тогда, когда вы наблюдаете за ними из кадра, в котором оба движутся. 2)}

$

Где $ X $ — постоянный ложный фактор, включающий расходы, константы и т. Д.

$ n $ — единичный вектор в направлении от источника (когда сила ушла) к цели (когда сила прибывает)

$ V_1 $ — это скорость источника (когда сила ушла) в нашем кадре, деленная на скорость света, обычно обозначаемая как $ \ beta $.$ V_1 $ перпендикулярно $ n $.

$ D $ — это расстояние от источника (когда сила ушла) до цели (когда сила ушла). $ D $ — это расстояние, на котором они неподвижны.

Это упрощенное выражение уравнения Лейнарда-Вейхерта, которое работает только тогда, когда скорость перпендикулярна $ n $, ускорение источника было нулевым, когда сила покинула источник, и цель не имеет другого ускорения, когда сила прибывает.

Поскольку мы выбрали кадр, в котором движутся источник и цель, расстояние от источника, когда сила уходит, от цели, когда сила прибывает, отличается.2)} = X \ frac {\ overline {n-v}} {D} $ Член $ \ overline {n-v} $ поворачивает вектор к исходному $ n $ в стационарном случае.

Уравнение L-W показывает, что в этом случае магнитная сила является ложным фактором, который дает нам ту же силу, когда мы назначаем систему отсчета с произвольной постоянной скоростью. Я не показал, что это верно во всех случаях, и даже не показал вывод этих упрощенных случаев из общего уравнения. Но я утверждаю, что это правда.

Как заставить свет перемещаться на разные расстояния за одно и то же время с классическими рамками — это более сложная головоломка, и теория относительности дает решение.

электромагнетизм — Скорость вращения катушки в однородном магнитном поле при равновесии

Равновесие достигается, когда чистый крутящий момент на якоре равен нулю. Поскольку, как мы увидим ниже, невозможно добиться исчезновения чистого крутящего момента в течение длительного периода времени, мы будем искать ситуацию, когда крутящий момент , усредненный по времени , исчезает.

Закон силы Лоренца говорит нам, что для провода с вектором длины $ \ vec {l} $, несущего ток $ I $ в однородном магнитном поле $ \ vec {B} $, сила $ \ vec {F} $ равна

\ begin {Equation} F = I \, \ vec {l} \ times \ vec {B} \; \ end {Equation}

Отсюда следует, что сила крутящего момента $ \ tau $ на якоре с $ N $ витками и площадью поперечного сечения $ A $ равна

\ begin {уравнение} \ tau = N \, I \, A \, B \, \ sin \ theta \ end {уравнение}

где $ \ theta $ — направление между вектором нормали поперечного сечения якоря и магнитным полем.(Геометрия на самом деле немного сложнее, чем вы можете представить вначале — я рекомендую изучить диаграмму в нижней части этой страницы).

Теперь мы хотим знать, что произойдет, если мы возьмем среднее значение крутящего момента за некоторый интервал времени, охватывающий много оборотов двигателя. $ N $, $ A $ и $ B $ являются постоянными в этой задаче (хотя см. Обсуждение $ B $ в самом конце этого ответа), поэтому единственные величины, которые могут исчезнуть в среднем, — это $ \ langle Я \ rangle $ и $ \ langle \ sin \ theta \ rangle $.

Если у этого двигателя нет коммутатора, то крутящий момент будет переключать направления каждые полупериод, и впоследствии якорю будет трудно достичь какой-либо заметной скорости вообще. Я предполагаю, что у двигателя есть коммутатор, который переключает ток каждые полупериод. В этом случае $ \ langle \ sin \ theta \ rangle \ not = 0 $, поэтому в состоянии равновесия мы должны иметь $ \ langle I \ rangle = 0 $.

Для достижения $ \ langle I \ rangle = 0 $ необходимо, чтобы $ \ langle \ epsilon_ \ mathrm {net} \ rangle = 0 $, где $ \ epsilon_ \ mathrm {net} $ — это чистая ЭДС. и содержит вклады от разности потенциалов 24 В и магнитной индукции из-за вращения якоря (но для получения более подробной информации см. последнее примечание в конце ответа).Как уже указывалось в постановке вопроса,

\ begin {уравнение} \ epsilon_ \ mathrm {net} = 24 \ mathrm {\; V} — N \, A \, B \ sin \ theta \ frac {d \ theta} {dt} \ end {Equation}

, а затем взяв среднее значение по времени для достижения равновесия,

\ begin {уравнение} \ langle \ sin \ theta \ frac {d \ theta} {dt} \ rangle = \ frac {24 \ mathrm {\; V}} {N \, A \, B} \ end {Equation}

Строго говоря, $ \ langle \ sin \ theta \ frac {d \ theta} {dt} \ rangle \ not = \ langle \ sin \ theta \ rangle \ langle \ frac {d \ theta} {dt} \ rangle $, потому что $ \ frac {d \ theta} {dt} $ будет меняться со временем в зависимости от $ \ sin \ theta $ (важно помнить, что на самом деле мы усредняем по времени, а не напрямую по $ \ theta $).Тогда оценка снизу для $ \ frac {d \ theta} {dt} $ — это ответ, приведенный в конце книги. Более правильный ответ был бы больше. В приближении, что $ \ frac {d \ theta} {dt} $ является постоянной во времени (на самом деле это не так, но, возможно, довольно хорошее приближение), тогда ответ будет умножен на множитель $ \ pi / 2 \ приблизительно 1.57 $.

Таким образом, я думаю, что вопрос был плохо сформулирован, и в данном случае вам не следует беспокоиться о точном совпадении ответа в книге.

Существует дополнительное магнитное поле, создаваемое током, протекающим через провода якоря.Поскольку ток изменяется во времени, поток этого поля через якорь будет изменяться во времени. Этот дополнительный изменяющийся магнитный поток в принципе должен влиять как на ЭДС в цепи, так и на крутящий момент на якоре.

Однако эти эффекты будут незначительными, если магнитное поле, создаваемое катушками, будет незначительным по сравнению с уже существующим магнитным полем 0,33 Тесла. Я призываю вас рассчитать, каков примерно максимальный ток, который может протекать по проводам, прежде чем предположение, что соответствующее поле незначительно, выйдет из строя (я проверил, и ток действительно должен стать довольно большим, но вы все равно должны проверить себя) .

Закон Фарадея

Закон Фарадея

Далее: Закон Ленца
Up: Магнитная индукция
Предыдущий: Магнитная индукция

Явление магнитной индукции играет решающую роль в
три очень полезных электрических устройства: электрогенератор , электрогенератор
двигатель
и трансформатор . Без этих устройств современная жизнь была бы
невозможно в его нынешнем виде. Магнитная индукция была открыта в 1830 г.
Английский физик Майкл Фарадей.Американский физик Джозеф Генри
независимо друг от друга сделал то же открытие примерно в одно и то же время. Оба
физиков заинтриговал тот факт, что электрический ток, протекающий вокруг
цепь может генерировать магнитное поле. Наверняка, рассуждали они, если электрический
ток может генерировать магнитное поле, тогда магнитное поле должно каким-то образом быть способным
генерировать электрический ток.
Однако потребовалось много лет бесплодных экспериментов.
прежде, чем они смогли найти необходимый ингредиент, который позволяет
магнитное поле для генерации электрического тока.Этот
ингредиент изменение времени .

Рассмотрим плоскую петлю из токопроводящего провода соответствующей площади поперечного сечения.
Поместим эту петлю в магнитное поле, напряженность которого приблизительно равна
равномерный по всей длине петли. Предположим, что направление
магнитное поле образует угол с нормальным направлением к
петля. Магнитный поток через петлю равен
определяется как произведение площади петли и составляющей
магнитное поле, перпендикулярное петле.Таким образом,

(191)



Если цикл оборачивается вокруг себя раз (, т. Е. , если цикл
имеет витков ) то магнитный поток через петлю просто
умножить на магнитный поток на один виток:

(192)



Наконец, если магнитное поле неоднородно по петле или петля не
лежать в одной плоскости,
тогда мы должны оценить
магнитный поток как поверхностный интеграл

(193)



Вот какая-то поверхность, к которой прикреплена.Если петля имеет витки, то поток в несколько раз превышает указанное выше значение.
Единица измерения магнитного потока в системе СИ — вебер (Вб). Одна тесла эквивалентна
один вебер на квадратный метр:

(194)


Фарадей обнаружил, что если магнитное поле проходит через петлю из проволоки
изменяется во времени , после чего вокруг контура наводится ЭДС.
Фарадей смог наблюдать этот эффект, потому что ЭДС вызывает
ток, циркулирующий в петле.Фарадей обнаружил, что величина
ЭДС прямо пропорциональна скорости изменения магнитного поля во времени.
Он также обнаружил, что ЭДС генерируется, когда петля провода перемещается на
из области низкой напряженности магнитного поля в область высокой напряженности магнитного поля, и наоборот . ЭДС прямо пропорциональна
скорость, с которой петля перемещается между двумя областями. Наконец,
Фарадей обнаружил, что ЭДС генерируется вокруг контура, который поворачивает на
в однородном магнитном поле постоянной напряженности.В этом случае ЭДС
прямо пропорциональна скорости вращения петли.
Фарадей в конце концов
в состоянии предложить единый
закон, который мог объяснить все его многочисленные и разнообразные наблюдения. Этот закон, известный как
Закон магнитной индукции Фарадея , выглядит следующим образом:

ЭДС, индуцированная в цепи, пропорциональна скорости изменения во времени
магнитный поток, связывающий эту цепь.


Единицы СИ были зафиксированы таким образом, чтобы константа пропорциональности в этом
закон единицы .Таким образом, если магнитный поток через цепь изменяется
на сумму во временном интервале
тогда генерируемая в цепи ЭДС равна

(195)


Есть много разных способов, которыми магнитный поток, связывающий
электрическая цепь может
изменять. Может измениться либо напряженность магнитного поля, либо направление магнитного поля.
поле может измениться, или положение цепи может измениться, или форма
цепь может измениться, или ориентация цепи может измениться.Закон Фарадея гласит, что все эти способы
полностью эквивалент по генерации ЭДС вокруг
цепь касается.


Далее: Закон Ленца
Up: Магнитная индукция
Предыдущий: Магнитная индукция

Ричард Фицпатрик
2007-07-14

Motional Emf — Университетская физика, том 2

Цели обучения

К концу этого раздела вы сможете:

  • Определить величину наведенной ЭДС в проводе, движущемся с постоянной скоростью через магнитное поле.
  • Обсудите примеры, в которых используется ЭДС движения, например, рельсовая пушка и привязанный спутник.

Магнитный поток зависит от трех факторов: силы магнитного поля, площади, через которую проходят силовые линии, и ориентации поля с площадью поверхности.Если какая-либо из этих величин изменяется, происходит соответствующее изменение магнитного потока. До сих пор мы рассматривали только изменения потока из-за изменяющегося поля. Теперь мы рассмотрим другую возможность: изменение области, через которую проходят силовые линии, включая изменение ориентации области.

Два примера этого типа изменения потока представлены на (Рисунок). В части (а) поток через прямоугольную петлю увеличивается по мере того, как она движется в магнитное поле, а в части (b) поток через вращающуюся катушку изменяется в зависимости от угла.

(a) Магнитный поток изменяется, когда петля движется в магнитное поле; (б) магнитный поток изменяется при вращении петли в магнитном поле.

Интересно отметить, что то, что мы воспринимаем как причину определенного изменения потока, на самом деле зависит от выбранной нами системы отсчета. Например, если вы находитесь в состоянии покоя относительно движущихся катушек на (Рисунок), вы увидите, что поток изменяется из-за изменения магнитного поля — в части (а) поле перемещается слева направо в вашей системе отсчета, и в части (б) поле вращается.Часто можно описать изменение магнитного потока через катушку, которая движется в одной конкретной системе отсчета, в терминах изменяющегося магнитного поля во второй системе отсчета, где катушка неподвижна. Однако вопросы системы отсчета, связанные с магнитным потоком, выходят за рамки этого учебника. Мы избежим таких сложностей, всегда работая в кадре в состоянии покоя относительно лаборатории и объясняя вариации потока как следствие либо изменяющегося поля, либо изменяющейся области.

Теперь давайте посмотрим на проводящий стержень, включенный в цепь, изменяющую магнитный поток.Площадь, ограниченная схемой «MNOP» (Рисунок), составляет лк x и перпендикулярна магнитному полю, поэтому мы можем упростить интеграцию (Рисунок) в умножение магнитного поля и площади. Следовательно, магнитный поток через открытую поверхность составляет

.

Поскольку B и l постоянны, а скорость стержня равна, мы можем теперь переформулировать закон Фарадея (рисунок) для величины ЭДС в единицах движущегося проводящего стержня как

Ток, наведенный в цепи, равен ЭДС, деленной на сопротивление, или

Кроме того, направление индуцированной ЭДС удовлетворяет закону Ленца, что вы можете проверить, посмотрев на рисунок.

Этот расчет ЭДС, вызванной движением, не ограничивается перемещением стержня по проводящим рельсам. В качестве отправной точки можно показать, что справедливо для любого изменения магнитного потока, вызванного движением проводника. Мы видели в законе Фарадея, что ЭДС, индуцированная изменяющимся во времени магнитным полем, подчиняется той же зависимости, которая является законом Фарадея. Таким образом, закон Фарадея выполняется для всех изменений магнитного потока , независимо от того, вызваны ли они изменяющимся магнитным полем, движением или их комбинацией.

Проводящий стержень перемещается вправо с постоянной скоростью. Результирующее изменение магнитного потока вызывает в цепи ток.

С точки зрения энергии производит мощность, а резистор ее рассеивает. Поскольку стержень движется с постоянной скоростью, приложенная сила должна уравновешивать магнитную силу на стержне, когда он пропускает наведенный ток I . Таким образом, произведенная мощность составляет

ед.

Рассеиваемая мощность

В соответствии с принципом сохранения энергии производимая и рассеиваемая мощности равны.

Этот принцип можно увидеть в работе рельсового пистолета. Рельсовая пушка — это электромагнитная пусковая установка для снарядов, в которой используется устройство, подобное (Рисунок), и схематично показано на (Рисунок). Проводящий стержень заменяется выстрелом или оружием. До сих пор мы слышали только о том, как движение вызывает ЭДС. В рельсовой пушке оптимальное отключение / уменьшение магнитного поля уменьшает поток между рельсами, вызывая протекание тока в стержне (якорь), удерживающем снаряд.Этот ток через якорь испытывает магнитную силу и продвигается вперед. Однако рельсовые пушки не используются широко в вооруженных силах из-за высокой стоимости производства и больших токов: для выработки энергии, достаточной для того, чтобы рельсовая пушка была эффективным оружием, требуется около миллиона ампер.

Ток через две рельсы движет токопроводящий снаряд вперед за счет создаваемой магнитной силы.

Мы можем вычислить ЭДС, индуцированную движением, с помощью закона Фарадея , даже когда фактически замкнутый контур отсутствует .Мы просто представляем замкнутую область, граница которой включает движущийся проводник, вычисляем, а затем находим ЭДС по закону Фарадея. Например, мы можем позволить движущемуся стержню (Рисунок) быть одной стороной воображаемой прямоугольной области, представленной пунктирными линиями. Площадь прямоугольника составляет лк , поэтому магнитный поток через него равен. Дифференцируя это уравнение, получаем

, что соответствует разности потенциалов между концами стержня, которую мы определили ранее.

С показанным воображаемым прямоугольником мы можем использовать закон Фарадея для расчета наведенной ЭДС в движущемся стержне.

ЭДС движения в слабом магнитном поле Земли обычно не очень велики, иначе мы могли бы заметить напряжение на металлических стержнях, таких как отвертка, во время обычных движений. Например, простой расчет ЭДС движения стержня длиной 1,0 м, движущегося со скоростью 3,0 м / с перпендикулярно полю Земли, дает

Это небольшое значение согласуется с опытом.Однако есть впечатляющее исключение. В 1992 и 1996 годах с космическим челноком были предприняты попытки создать большие двигательные ЭДС. Привязанный спутник должен был быть выпущен на проводе длиной 20 км, как показано на (Рисунок), для создания ЭДС 5 кВ за счет движения с орбитальной скоростью через поле Земли. Эту ЭДС можно было бы использовать для преобразования некоторой кинетической и потенциальной энергии шаттла в электрическую, если бы можно было создать полную цепь. Чтобы замкнуть цепь, неподвижная ионосфера должна была обеспечить обратный путь, по которому мог течь ток.(Ионосфера — это разреженная и частично ионизированная атмосфера на орбитальных высотах. Она является проводящей из-за ионизации. Ионосфера выполняет ту же функцию, что и неподвижные рельсы и соединительный резистор на (Рисунок), без которых не было бы полной цепи.) Затягивание тока в кабеле из-за магнитной силы выполняет работу, которая уменьшает кинетическую и потенциальную энергию шаттла и позволяет преобразовывать ее в электрическую энергию. Оба теста не увенчались успехом. В первом случае кабель завис, и его можно было протянуть только на пару сотен метров; во втором трос оборвался при почти полном растяжении.(Рисунок) указывает на выполнимость в принципе.

ЭДС движения как преобразование электроэнергии для космического корабля многоразового использования была мотивацией для эксперимента с привязанным спутником. Было предсказано, что ЭДС 5 кВ будет индуцироваться в 20-километровом тросе при движении с орбитальной скоростью в магнитном поле Земли. Цепь замыкается обратным трактом через неподвижную ионосферу.

Металлический стержень, вращающийся в магнитном поле На части (a) (Рисунок) показан металлический стержень OS , который вращается в горизонтальной плоскости вокруг точки O .Стержень скользит по проволоке, которая образует дугу окружности PST радиусом r . Система находится в постоянном магнитном поле, направленном за пределы страницы. (a) Если вы вращаете стержень с постоянной угловой скоростью, каков ток I в замкнутом контуре OPSO ? Предположим, что резистор R обеспечивает все сопротивление в замкнутом контуре. (b) Рассчитайте работу за единицу времени, которую вы делаете при вращении стержня, и покажите, что она равна мощности, рассеиваемой в резисторе.

Стратегия Магнитный поток — это магнитное поле, умноженное на площадь четверти круга. При нахождении ЭДС по закону Фарадея все переменные постоянны во времени, но для расчета работы в единицу времени мы знаем, что это связано с крутящим моментом. умножить на угловую скорость. Крутящий момент рассчитывается исходя из силы, действующей на стержень, и ее интегрирования по длине стержня.

Решение

  1. Исходя из геометрии, площадь контура OPSO равна Следовательно, магнитный поток через контур равен

    Дифференцируя по времени и использованию, получаем

    Деление на сопротивление контура R дает величину индуцированного тока

    По мере увеличения увеличивается и поток через петлю. Чтобы противодействовать этому увеличению, магнитное поле из-за индуцированного тока должно быть направлено на страницу в области, заключенной в петлю.Следовательно, как показано в части (b) (Рисунок), ток циркулирует по часовой стрелке.

  2. Вы вращаете стержень, прилагая к нему крутящий момент. Поскольку стержень вращается с постоянной угловой скоростью, этот крутящий момент равен и противоположен крутящему моменту, приложенному к току в стержне исходным магнитным полем. Магнитная сила на бесконечно малом сегменте длиной dx , показанном в части (c) (Рисунок), равна магнитному моменту на этом сегменте

    Чистый магнитный крутящий момент на стержне будет

    Крутящий момент, который вы прилагаете к стержню, равен и противоположен ему, а работа, которую вы выполняете, когда стержень поворачивается на угол, равна Следовательно, работа на единицу времени, которую вы выполняете на стержне, равна

    , где мы заменили I .Мощность, рассеиваемая в резисторе, может быть записана как

    Следовательно, мы видим, что

    Следовательно, мощность, рассеиваемая в резисторе, равна работе, совершаемой в единицу времени при вращении стержня.

Значение. Альтернативный способ взглянуть на индуцированную ЭДС из закона Фарадея — интегрировать в пространстве, а не во времени. Решение, однако, будет таким же. Двигательная ЭДС

Скорость может быть записана как угловая скорость, умноженная на радиус, а дифференциальная длина — как dr .Следовательно,

, это то же самое решение, что и раньше.

Проверьте свое понимание Стержень длиной 10 см движется со скоростью 10 м / с перпендикулярно через магнитное поле напряжением 1,5 Тл. Какая разница потенциалов между концами стержня?

Сводка

  • Связь между наведенной ЭДС в проводе, движущемся с постоянной скоростью v через магнитное поле B , задается формулой
  • Индуцированная ЭДС по закону Фарадея создается двигательной ЭДС, которая противодействует изменению потока.

Концептуальные вопросы

Барный магнит падает под действием силы тяжести вдоль оси длинной медной трубки. Если сопротивление воздуха незначительно, появится ли сила, препятствующая спуску магнита? Если да, достигнет ли магнит предельной скорости?

Вокруг географического Северного полюса (или южного магнитного полюса) магнитное поле Земли почти вертикальное. Если в этой области самолет летит на север, какая сторона крыла заряжена положительно, а какая отрицательно?

Положительные заряды на крыльях будут к западу или слева от пилота, а отрицательные заряды будут тянуться к востоку или справа от пилота.Таким образом, кончики левых крыльев будут положительными, а кончики правых — отрицательными.

Проволочная петля движется поступательно (без вращения) в однородном магнитном поле. В шлейфе наведена ЭДС?

Глоссарий

двигательная ЭДС
напряжение, создаваемое движением проводящего провода в магнитном поле

Закон Фарадея

Концепция закона Фарадея состоит в том, что любое изменение магнитной среды катушки с проволокой вызывает «индуцирование» в катушке напряжения (ЭДС).Независимо от того, как производится изменение, напряжение будет генерироваться. Изменение может быть произведено изменением напряженности магнитного поля, перемещением магнита к катушке или от нее, перемещением катушки в магнитное поле или из него, вращением катушки относительно магнита и т. Д.

Слева вверху на иллюстрации две катушки пронизаны изменяющимся магнитным полем. Магнитный поток F определяется как F = BA, где B — магнитное поле или среднее магнитное поле, а A — площадь, перпендикулярная магнитному полю.Обратите внимание, что для данной скорости изменения потока через катушку генерируемое напряжение пропорционально количеству витков N, через которые проходит поток. Этот пример относится к работе трансформаторов, где магнитный поток обычно следует за железным сердечником от первичной обмотки ко вторичной обмотке и генерирует вторичное напряжение, пропорциональное количеству витков вторичной обмотки.

По часовой стрелке второй пример показывает напряжение, генерируемое при перемещении катушки в магнитное поле.Иногда это называют «ЭДС движения», и она пропорциональна скорости, с которой катушка перемещается в магнитное поле. Эта скорость может быть выражена через скорость изменения области, находящейся в магнитном поле.

Следующий пример — это стандартная геометрия генератора переменного тока, в которой катушка с проволокой вращается в магнитном поле. Вращение изменяет перпендикулярную площадь катушки по отношению к магнитному полю и генерирует напряжение, пропорциональное мгновенной скорости изменения магнитного потока.При постоянной скорости вращения генерируемое напряжение является синусоидальным.

Последний пример показывает, что напряжение можно генерировать, перемещая магнит к катушке с проволокой или от нее. При постоянной площади изменяющееся магнитное поле вызывает генерируемое напряжение. Направление или «смысл» генерируемого напряжения таковы, что любой результирующий ток создает магнитное поле, противодействующее изменению магнитного поля, которое его создало. Это значение знака минус в законе Фарадея, и это называется законом Ленца.

33 Измерение скорости с помощью электромагнитной индукции

Электромагнитные регистраторы скорости продолжают оставаться популярными для измерения движения судна в воде. Этот тип журнала использует хорошо задокументированный принцип Майкла Фарадея измерения потока жидкости мимо датчика с помощью электромагнитной индукции.

Операция основана на принципе, что любой проводник, который перемещается поперек магнитного поля, индуцирует в нем небольшую электродвижущую силу (например,м.ф.). В качестве альтернативы e.m.f. также будет индуцироваться, если проводник остается неподвижным, а магнитное поле перемещается относительно него. Если предположить, что магнитное поле остается постоянным, амплитуда наведенной ЭДС. будет прямо пропорционально скорости движения.

При практической установке постоянная э.д.с. в проводнике (морская вода, протекающая мимо датчика), и в коллекторе индуцируется незначительный ток, пропорциональный относительной скорости. Магнитное поле, создаваемое в морской воде, создается соленоидом, который может выходить в воду или устанавливаться заподлицо с корпусом.По мере движения судна морская вода (проводник), протекающая через магнитное поле, имеет небольшую ЭДС. побудил в это. Эту минутную э.д.с., амплитуда которой зависит от скорости перерезания магнитных силовых линий, регистрируется двумя небольшими электродами, установленными во внешнем корпусе датчика.

На рис. 3.7 показан соленоид, создающий магнитное поле, и проводник, соединенный в виде петли, способной перемещаться под прямым углом к ​​полю. Если проводник перемещать в указанном направлении, в проводе будет индуцироваться крошечный ток и небольшой e.м.ф. производится поперек него. В случае электромагнитного каротажа скорости проводником является морская вода, проходящая через магнитное поле. Правило Флеминга показывает, что сгенерированная ЭДС. находится под прямым углом к ​​магнитному полю (H). Индуцированный ток, протекающий в проводнике, позволяет определить ЭДС. на счетчике. Если предположить, что ток включения соленоида составляет постоянный ток, индуцированная ЭДС. это plv, где p = индуцированное магнитное поле, l = длина проводника и v = скорость проводника.

p примерно равно H, напряженности магнитного поля. Следовательно, e.m.f. = Hlv при отсутствии потерь в цепи.

Для уменьшения эффектов электролиза и усиления наведенной ЭДС. проще, переменный ток используется для создания магнитного поля. Напряженность магнитного поля H теперь становится Hmsinwt, а наведенная э.д.с. это: Hmlvsinwt. Если напряженность магнитного поля и длина проводника остаются постоянными, то э.д.с. — скорость.

Рисунок 3.8 показано, что изменения ЭДС, вызванные изменениями скорости, образуют линейный график и, таким образом, линейную индикацию скорости судна. Э.д.с. полученный таким образом очень маленький

Рис. 3.7. Эффект перемещения проводника в магнитном поле.

, но, если требуется, может быть увеличен за счет увеличения тока включения или количества витков провода на соленоиде. Следует отметить следующие моменты.

• Переменный ток При подаче питания на соленоид возникает индуктивный контакт между катушкой и проводами, по которым передается сигнал.Это, в свою очередь, приводит к «нулевой» ошибке, которую необходимо компенсировать, «откладывая» нулевую установку индикатора при калибровке.

• Индуцированная э.д.с. очень мала (для разумных амплитуд тока возбуждения), обычно 100 пВ на узел.

• Индуцированная э.д.с. и, следовательно, индикация скорости будет изменяться в зависимости от проводимости воды.

• Устройство измеряет скорость воды, протекающей мимо корпуса корабля. Этот поток может меняться из-за нелинейности конструкции корпуса.

• Океанские течения могут вносить ошибки.

• Качка и качение влияют на соотношение между скоростью относительно воды и корпусом. Ошибка из-за этого эффекта может быть компенсирована снижением чувствительности приемника. Это достигается с помощью схемы синхронизации CR с большой постоянной времени для гашения колебательного эффекта.

• Точность обычно составляет 0,1% от используемой дальности в продольном и продольном направлениях, и приблизительно 2% при отражении от берега.

Рисунок 3.9 показан типичный разрез датчика, показывающий соленоид и считывающие электроды. Система преобразования скорости показана на рисунке 3.10.

Рисунок 3.9 Конструктивные детали датчика электромагнитного каротажа.
Рисунок 3.10 Э.М. система быстрого перевода логов.

Описание системы скоростного перевода

Напряжение малого сигнала скорости от датчика, e.m.f.1, подается на дифференциальный трансформатор, где оно сравнивается с опорным напряжением e.m.f.2, создаваемое потенциометром на входе переменного тока. поставлять. Разность потенциалов, возникающая на опорном резисторе, обеспечивает ток питания для соленоида в датчике.

Если напряжение сигнала э.м.д. 1. отличается от эталонного напряжения ЭДС 2. сигнал ошибки по напряжению 8 э.м. производится. Это напряжение ошибки подается на усилитель сигнала скорости, где оно усиливается для выработки достаточной мощности для привода серводвигателя. Сервопривод, в свою очередь, будет показывать скорость через механическую связь на индикаторе.Также с валом сервопривода соединен ползунок потенциометра скорости, который поворачивается в направлении уменьшения напряжения ошибки 8 э.м.д. Когда напряжение этой ошибки падает до нуля, сервопривод перестает вращаться. Индикатор скорости остается неподвижным до следующей ошибки напряжения 8 э.м.д. производится. Каждый раз, когда создается напряжение ошибки, сервопривод поворачивается, чтобы устранить ошибку и, таким образом, уравновесить систему.

3.3.1 Практическая электромагнитная система регистрации скорости

Потенциал, развиваемый на электродах преобразователя, пропорционален напряженности магнитного поля (и, следовательно, току возбуждения) и скорости потока в объеме воды, на который влияет поле.Напряженность магнитного поля никоим образом не стабилизируется по отношению к каким-либо изменениям в основном напряжении, температуре и т. Д. На корабле, но, эффективно сравнивая ток возбуждения с напряжением на электродах, их соотношение позволяет измерить скорость корабля.

Входной трансформатор T1 (показан на рисунке 3.11) обладает очень высокой индуктивностью и понижающим коэффициентом 5: 1. Это приводит к входному импедансу, наблюдаемому с помощью измерительных электродов, приближающемуся к 20 Mft, что по сравнению с импедансом, представленным соленой водой, можно рассматривать как разомкнутую цепь.Следовательно, изменения солености не влияют на измеренное напряжение и результирующую индикацию скорости. Цепочка переключаемых резисторов (R1 / R5) устанавливает коэффициент усиления всего усилителя вместе с цепью резисторов (R6 / R10), которая регулирует амплитуду сигнала обратной связи.

Выход IC1 связан через IC2, который из-за емкостной обратной связи (не показана) гарантирует, что схема имеет нулевой сдвиг фазы от T1 до T2 к демодулятору. Демодуляция осуществляется посредством TR1 / TR2, которые в свою очередь переключаются с a.c. опорное напряжение, получаемое от тороидального трансформатора, контролирующего ток включения преобразователя. При синхронном возбуждении TR1 / TR2 фазовое соотношение напряжения, обнаруживаемого электродами, определяет полярность демодулированного сигнала. Фазировка 0 ° и 180 ° дает положительный или отрицательный компонент; 90 ° и 270 ° не дают выхода и, следовательно, полностью отклоняют такие фазово-квадратурные сигналы. Демодулированный сигнал подается на интегратор Миллера IC3, который, в свою очередь, приводит в действие генератор тока.Ретрансляторы скорости питаются от этого источника.

Работа контура

При отсутствии движения судна будет нулевой сигнал на входе в IC1 и, следовательно, не будет сигнала на входе микросхемы умножителя. На входе IC1 сигнал обратной связи не формируется. Когда судно движется вперед, слабый сигнал, подаваемый на IC1, обрабатывается в электронном блоке для создания тока, протекающего через ретрансляторы скорости и умножитель. Теперь существует выходной сигнал умножителя, пропорциональный току ретранслятора скорости и опорному напряжению, создаваемому тороидальным трансформатором, контролирующим ток включения преобразователя.Переменный ток от умножителя подается обратно на IC1 последовательно и сдвинуто по фазе на 180 ° с вторичным малым сигналом T1. Это переменный ток. сигнал растет медленно и в конечном итоге с постоянной времени демодулятора становится равным сигналу p.d.

SC Переключатель диапазонов

SC Переключатель диапазонов

Удаленные повторители

Преобразователь

Рисунок 3.11 Упрощенная схема ЭМ. бревно. (Воспроизведено с разрешения Thomas Walker and Son Ltd.)

Преобразователь

Удаленные повторители

200 импульсов на морскую милю 3100 импульсов на морскую милю 1 импульс на морскую милю

Рисунок 3.11 Упрощенная схема ЭМ. бревно. (Воспроизведено с разрешения Thomas Walker and Son Ltd.)

разработан в T1. В это время результирующий сигнал, подаваемый на IC1, падает до нуля, и поэтому выходной сигнал демодулятора остается на постоянном уровне. Любое дальнейшее изменение скорости приводит к дисбалансу вторичной обмотки Т1, в результате чего возникает переменный ток. сигнал на IC1. В результате выходной сигнал демодулятора увеличивается или уменьшается (быстрее или медленнее скорость корабля) до тех пор, пока не будет восстановлено состояние баланса.Репитеры скорости укажут соответствующее изменение скорости.

Дистанционная интеграция

Ток скорости пропускается через резистивную сеть на дистанционной интеграционной плате, чтобы можно было создать пропорциональное напряжение для интеграции. Выходной сигнал этой платы представляет собой последовательность импульсов, частота которой пропорциональна указанной скорости. Импульсы 10 мс подаются на плату управления реле, которая содержит необходимую логику для выдачи следующих выходных сигналов: 200 импульсов на морскую милю, 100 импульсов на морскую милю и 1 импульс на морскую милю.

Прочтите здесь: 34 Измерение скорости с использованием методов акустической корреляции

Была ли эта статья полезной?

.

Добавить комментарий

Ваш адрес email не будет опубликован. Обязательные поля помечены *